Sie sind auf Seite 1von 61

Sim

acr ul

ipo ot

AR EN

010 2 M

CTO Mjico

nmero: 2

EXAMEN ENARM 2/10

1.

Pregunta vinculada a la imagen n 1. Una paciente de 58 aos de edad con antecedentes de amigdalitis de repeticin en la infancia por lo que nalmente fue amigdalectomizada a la edad de 3 aos, acude a consulta por disnea progresiva con esfuerzos moderados de 3 meses de evolucin. La PA es de 125/75 mmHg, el pulso es irregular a 115 lpm, y en la exploracin fsica se detectan escasos crepitantes en ambas bases, y en pex se ausculta un chasquido protodiastlico seguido de un soplo diastlico de baja frecuencia con disminucin en la intensidad del primer ruido. El ECG muestra brilacin auricular con respuesta ventricular algo rpida sin anomalas en el QRS ni en la repolarizacin. Se realiza una radiografa de trax posteroanterior (IMAGEN). Respecto a esta paciente seale la opcin falsa: 1. Es probable que la entrada en brilacin auricular haya ocasionado el deterioro clnico actual. 2. El soplo probablemente muestre un refuerzo presistlico (telediastlico). 3. La ebre reumtica es la causa ms frecuente de su valvulopata. 4. Pueden existir variaciones en la intensidad del primer ruido. 5. Cuanto ms prximo se encuentre el chasquido respecto al segundo ruido nos indica mayor severidad de su valvulopata.

4.

Pregunta vinculada a la imagen n 2. Un paciente de 58 aos, hipertenso y dislipmico acude a urgencias por dolor torcico opresivo irradiado a brazo izquierdo que comenz estando en reposo de 2 horas de evolucin, con intenso cortejo vegetativo y disnea progresiva asociados. A su llegada la PA es 80/40 mmHg y satura al 89%. En la exploracin destaca la presencia de crepitacin en ambos campos pulmonares inferiores y tercer ruido izquierdo. Se realiza un ECG con los siguientes hallazgos (IMAGEN). Respecto a este paciente seale la respuesta incorrecta: 1. Podra tratarse de un infarto agudo de miocardio. 2. La imagen muestra una taquicardia ventricular probablemente por reentrada. 3. En esta situacin la angioplastia primaria (si est disponible) sera preferible a la brinolisis. 4. En caso de no disponer de programa de angioplastia primaria, estara indicada la brinolisis. 5. Debe iniciarse tratamiento con aspirina.

5.

Pregunta vinculada a la imagen n 2. Seale cul de los siguientes hallazgos ecocardiogrcos permitira establecer el diagnstico de infarto agudo de miocardio en el paciente de la pregunta anterior: 1. Estenosis artica con gradiente medio de 60 mmHg. 2. Insuciencia mitral leve. 3. Aquinesia de la cara anterior y septo. 4. Derrame pericrdico moderado. 5. Alteracin de la relajacin.

2.

Pregunta vinculada a la imagen n 1. Respecto a la imagen radiolgica (IMAGEN) de la pregunta anterior seale la opcin incorrecta: 1. Es compatible con una estenosis mitral importante. 2. Se aprecian datos de crecimiento de la aurcula izquierda. 3. Existe una marcada elevacin anmala del hemidiafragma derecho. 4. La imagen no est correctamente centrada, lo que diculta su interpretacin. 5. Ambos senos costofrnicos estn libres de derrame.

6.

Pregunta vinculada a la imagen n 2. Respecto al ECG (IMAGEN) seale la opcin correcta. 1. Se trata de un bloqueo completo de rama derecha. 2. Muestra un patrn de preexcitacin ventricular. 3. Se trata de un infarto antiguo de miocardio inferior. 4. Existe bloqueo completo de la rama izquierda. 5. Existe un utter auricular.

3.

Pregunta vinculada a la imagen n 1. Respecto al manejo de esta paciente (IMAGEN), seale la opcin verdadera: 1. No precisa anticoagulacin crnica salvo que se planee una cardioversin de su brilacin auricular. 2. Los diurticos no son tiles para mejorar sus sntomas. 3. Los betabloqueantes estn contraindicados por estar en insuciencia cardiaca clnica actualmente. 4. El ecocardiograma no permitir conrmar el diagnstico y sern necesarias otras pruebas como la resonancia magntica. 5. La digoxina puede ser til en su tratamiento.

7.

Pregunta vinculada a la imagen n 3. Un paciente acude a consulta por disnea de grandes esfuerzos sin dolor torcico. Entre sus antecedentes destaca un consumo enlico excesivo durante una dcada. En la exploracin fsica se detecta un soplo sistlico en pex 2/6 irradiado a axila. La PA es de 110/60 mmHg y est taquicrdico a 103 lpm en sinusal, con PVY levemente elevada y edema bilateral en

-2-

EXAMEN ENARM 2/10

MMII. La RXTX posteroanterior se muestra en la imagen (IMAGEN). Cul de los siguientes le parece el diagnstico ms probable? 1. 2. 3. 4. 5. Miocardiopata Miocardiopata Miocardiopata Miocardiopata Miocardiopata recho. no compactada. dilatada. restrictiva. hipertrca. arritmognica de ventrculo de12.

1. Esclerosis endoscpica + colocacin de hemoclips + perfusin i.v. de IBP. 2. Esclerosis endoscpica + terlipresina i.v. 3. Ligadura endoscpica con bandas. 4. Ligadura endoscpica con bandas + somatostatina i.v. en perfusin continua. 5. Colocacin de hemoclips y perfusin de IBP durante 3 das por va intravenosa. Pregunta vinculada a la imagen n 6. Varn de 50 aos hipertenso y diabtico no insulindependiente que acude a urgencias por nauseas y vmitos alimentario-biliosos, distensin abdominal y dolor abdominal clico generalizado. En la analtica realizada en urgencias se observa: Hb 16,3 g/dl, VCM 86 , 16.500 leucocitos, plaquetas 325.000/mm3, INR 0,9, bilirrubina 1,2, AST 52, ALT 32, GGT 73, amilasa 46 UI/ ml, glucemia 86, urea 88, creatinina 1,6, sodio 149, potasio 4,3. Se realiza una Rx de abdomen que muestra la imagen adjunta. Cul de entre las siguientes sera la actitud MENOS adecuada? 1. 2. 3. 4. 5. 13. Solicitar una ecografa abdominal. Colocar sonda nasogstrica. Dieta absoluta. Reposicin hidroelectroltica. Pautara analgsicos.

8.

Pregunta vinculada a la imagen n 3. Cul de las siguientes opciones no est indicada en el manejo de este paciente? 1. 2. 3. 4. 5. Abstencin absoluta del consumo de alcohol. Antiagregacin crnica con aspirina. IECA o ARAII. Evitar en lo posible el empleo de AINE. Restriccin de sodio en la dieta.

9.

Pregunta vinculada a la imagen n 3. Respecto al tratamiento farmacolgico, cul de las siguientes medidas se ha relacionado con un incremento de mortalidad en estos pacientes? 1. 2. 3. 4. 5. Ibopamina. Ramipril. Carvedilol. Digoxina. Amiodarona.

Pregunta vinculada a la imagen n 6. Tras ligera mejora con el objeto de liar la etiologa, realiza una colonoscopia donde se observa una mucosa de recto normal y al llegar a sigma, abundantes divertculos rodeados por una mucosa intensamente hipermica, edematosa y con algunos restos de secrecin purulenta. Cual sera su actitud a continuacin? 1. Procedera a retirar el colonoscopio inmediatamente. 2. Continuara hasta ciego para descartar enfermedad de Crohn que justique ese patrn mucoso. 3. Tomara biopsias del fondo del divertculo para estudio microbiolgico. 4. Continuara por lo menos hasta ngulo esplnico para ver la extensin de la enfermedad. 5. Tratara de llegar hasta ngulo heptico dada la alta probabilidad de que se trate de una colitis isqumica cuyas lesiones ms graves se observaran en colon transverso.

10.

Pregunta vinculada a la imagen n 4. Sobre el procedimiento teraputico que se objetiva en la imagen seale la cierta: 1. Es el tratamiento de primera lnea en la prolaxis primaria de la hemorragia digestiva por varices esofgicas. 2. Su ecacia en la hemorragia activa es similar a la esclesosis endoscpica. 3. No es necesaria la repeticin del procedimiento para la erradicacin de las varices esofgicas. 4. La combinacin de este procedimiento, junto con la administracin de somatostatina intravenosa, es el tratamiento de eleccin de la hemorragia activa por varices esofgicas. 5. Es el tratamiento de primera lnea en la prolaxis secundaria de la hemorragia digestiva por varices esofgicas. 14.

Pregunta vinculada a la imagen n 7. Varn de 60 aos que desde hace 4 das presenta dolor y enrojecimiento en pierna derecha. Qu le parece verdadero respecto a la imagen? 1. Se observa una ppula, lesin tpica del liquen plano. 2. Se observa una placa, lesin tpica de la celulitis.

11.

Pregunta vinculada a la imagen n 5. Mujer de 57 aos que acude al servicio de urgencias por hematemesis. A su llegada palidez mucocutnea. PA 110/70 FC 75 lpm. Se realiza endoscopia urgente objetivndose (ver imagen). Con respecto a la patologa que se objetiva en la imagen seale el tratamiento de eleccin:

-3-

EXAMEN ENARM 2/10

3. Se observa una vescula, lesin tpica del herpes. 4. Se observa una ampolla, lesin tpica del pengoide ampolloso. 5. Se observa una lcera, lesin tpica del pioderma gangrenoso. 15. Pregunta vinculada a la imagen n 7. Respecto al paciente de la pregunta anterior, qu le parece falso: 19. 1. Si presentase signo de homans positivo, sera recomendable descartar una trombosis venosa profunda. 2. Si tuviera ebre, el diagnstico ms probable sera un eritema nodoso. 3. Es probable que exista aumento de temperatura a la palpacin. 4. Los hemocultivos suelen ser negativos. 5. Una erosin apenas perceptible puede haber sido el origen del cuadro. 16. Pregunta vinculada a la imagen n 8. Seale la verdadera acerca de la gura: 20. 1. 2. 3. 4. La lesin elemental predominante son las pstulas. La lesin elemental predominante son las vesculas. La lesin elemental predominante son las ppulas. La lesin elemental predominante son las excoriaciones. 5. La lesin elemental predominante son las seudovesculas. 17. Pregunta vinculada a la imagen n 8. El paciente es un varn de 40 aos que acude por estas lesiones de aparicin hace 3 das. Seale la falsa: 1. Si reere dolor, tendramos que pensar en un herpes zster. 2. Si reriera prurito y escozor, no podra descartarse un herpes simple. 3. Es poco probable que el paciente haya pasado la varicela de pequeo. 4. Sera recomendable un tratamiento tpico con cido fusdico. 5. En caso de tratar con antivirales, la va de eleccin sera la oral. 21. 18. Pregunta vinculada a la imagen n 9. Una paciente de 52 aos, sin antecedentes personales o familiares de inters, es remitida a consulta de endocrinologa por cuadro de varios meses de evolucin consistente en episodios autolimitados de intenso malestar, visin borrosa, cefalea, sudoracin, palidez y palpitaciones. Se realiza determinacin de metanefrina y normetanefrina en orina de 24 horas, encontrndose elevadas. Respecto a la prueba diagnstica de la imagen:

1. Es la primera prueba a realizar tras el diagnstico bioqumico. 2. Es obligada su realizacin en esta patologa. 3. Se puede realizar con I-123 o I-131 como istopo. 4. Es poco til para localizar patologa extrasuprarrenal. 5. La presencia de patologa tiroidea contraindica su realizacin. Pregunta vinculada a la imagen n 9. Respecto a la patologa que presenta la paciente del caso anterior, seale la verdadera: 1. Probablemente ser de carcter hereditario. 2. El tratamiento farmacolgico prequirrgico puede incluir antagonistas del calcio. 3. Es obligada la determinacin de calcitonina en este caso. 4. Es la causa ms frecuente de hipertensin arterial secundaria en la neurobromatosis tipo 1. 5. Se recomienda dieta hiposdica unos das antes de la ciruga. Pregunta vinculada a la imagen n 10. Una paciente de 85 aos, sin antecedentes familiares ni personales de inters, acude remitida por atencin primaria con la siguiente analtica hormonal: T4L 2,4 ng/dl (normal 0,8-2 ng/dl) y TSH indetectable (normal 0,4-4,0 mUI/L). La paciente presenta clnica de nerviosismo, palpitaciones y prdida de peso no cuanticada. A la exploracin se evidencia un ndulo tiroideo de unos 2,5 cm en LTD elstico y mvil a la deglucin. Se realiza una gammagrafa tiroidea que muestra los resultados de la imagen. Seale lo correcto: 1. Se trata de la causa ms frecuente de hipertiroidismo en adultos jvenes. 2. Suele presentar asociacin familiar. 3. La presencia de anticuerpos contra la peroxidasa tiroidea (Abs-TPO) es la norma en esta causa de hipertiroidismo. 4. Su aparicin es ms frecuente en reas decitarias en yodo que en pases yodo-sucientes. 5. Aparece con ms frecuencia en varones. Pregunta vinculada a la imagen n 10. Respecto al manejo de la patologa anterior, seale lo incorrecto: 1. Dadas las caractersticas gammagrcas y los datos a la exploracin, es necesario la realizacin de una puncin aspiracin con aguja na (PAAF) para descartar malignidad. 2. Para el manejo de los sntomas puede ser til la administracin de atenolol a dosis de 25-50 mg/da.

-4-

EXAMEN ENARM 2/10

3. Sera conveniente la administracin de antitiroideos para obtener un estado de normofuncin tiroidea antes del tratamiento denitivo. 4. Como tratamiento denitivo se puede utilizar I-131 o ciruga. 5. Unos das antes de la administracin del I-131 se deben suspender los antitiroideos para que no exista interaccin con la prueba. 22. Pregunta vinculada a la imagen n 11. Respecto a la patologa que se muestra en la imagen, seale lo falso: 1. La administracin de yodo en estos pacientes puede producir hipertiroidismo. 2. Disnea, estridor, disfagia y tos pueden estar presentes. 3. La teora etiopatognica clsica sugiere que el crecimiento glandular es la respuesta a cualquier factor que impida una correcta sntesis de hormonas tiroideas. 4. El dcit de yodo y los bocigenos pueden estar implicados. 5. El TAC es muy til visualizar el compromiso traqueal, pero valora mal la parte posterior y zona retroesternal. 23. Pregunta vinculada a la imagen n 11.

25.

Pregunta vinculada a la imagen n 13. Paciente de 30 aos, primigesta. Tras un periodo de dilatacin normal alcanza dilatacin completa. Tras 30 minutos de expulsivo, con monitorizacin normal, comienza con el registro que se muestra en la imagen. La exploracin muestra una presentacin ceflica, OIIA, III plano de Hodge. Cul sera su actitud? 1. 2. 3. 4. 5. Esperar parto espontneo. Realizar un forceps. Aplicar una ventosa obsttrica. Realizar unas esptulas. Realizar una cesrea.

26.

Pregunta vinculada a la imagen n 14. En el estudio de morfologa de clulas sanguneas de un paciente con anemia se aprecian los hallazgos que muestra la imagen. Qu prueba complementaria solicitara a continuacin? 1. 2. 3. 4. 5. Prueba de Coombs. Sideremia, transferrina y ferritina. Electroforesis de hemoglobinas. Niveles sricos de folato y B12. Citogentica de mdula sea.

27. Respecto al manejo de la patologa de la imagen, seale lo correcto: 1. La administracin de levotiroxina en dosis supresoras es el tratamiento de eleccin cuando existe sintomatologa compresiva. 2. Aunque el tratamiento de eleccin es la ciruga, la administracin de I-131 puede ser una opcin razonable en pacientes sintomticos de mayor edad y malos candidatos a la ciruga. 3. La traqueomalacia por compresin durante la ciruga, es la complicacin posquirrgica ms frecuente. 4. No es necesario realizar determinacin de TSH ni otros estudios antes de la ciruga. 5. La ciruga de eleccin suele ser una itsmectoma. 24. Pregunta vinculada a la imagen n 12. Secundigesta de 28 aos, con un parto eutcico previo, gestante de 39 semanas. El embarazo ha cursado con normalidad. Durante el periodo de dilatacin se obtiene el registro que se muestra en la imagen. Condiciones obsttricas: ceflica, lquido amnitico claro, 5 cm, II plano Hogde. Conducta a seguir: 1. 2. 3. 4. 5. Cesrea. Colocar en decbito lateral izquierdo. Realizar microtoma fetal. Administrar betamimticos. Retirar goteo oxitocina.

Pregunta vinculada a la imagen n 14. El laboratorio le informa que el resultado de la prueba que usted solicit se demorar debido a problemas tcnicos. Qu tratamiento administrara al paciente durante este tiempo? 1. 2. 3. 4. 5. Prednisona va oral. Sulfato ferroso oral. Eritropoyetina. B12 parenteral y cido flico oral. Mesilato de imatinib oral.

28.

Pregunta vinculada a la imagen n 14. Para el estudio de la anemia del paciente, seale cul de las siguientes pruebas complementarias ser necesaria: 1. 2. 3. 4. 5. Sangre oculta en heces. TC abdominal en busca de esplenomegalia. Anticuerpos antifactor intrnseco. Anticuerpos antinucleares. Radiografa torcica en busca de posible neoplasia pulmonar.

29.

Pregunta vinculada a la imagen n 15. Mujer de 30 aos diagnosticada de LES, en tratamiento con AINE, por artritis de rodilla. Acude al servicio de Urgencias por hematemesis. PA 120/40 FC 60. Hb 14,5. Se realiza una endoscopia urgente observndose (ver imagen).

-5-

EXAMEN ENARM 2/10

Qu binomio diagnstico-teraputico considera correcto? 1. lcera duodenal Forrest Ia Esclerosis endoscpica + perfusin i.v. de IBP. 2. lcera duodenal Forrest Ib Esclerosis endoscpica + hemoclip + perfusin i.v. de IBP. 3. Hemorragia por varices Esclerosis + somatostatina i.v. 4. lcera duodenal Forrest III IBP va oral y alta hospitalaria. 5. lcera duodenal Forrest IIa Esclerosis endoscpica e IBP va oral. 30. Pregunta vinculada a la imagen n 16. En un paciente con una hernia hiatal paraesofgica con clnica de reujo gastroesofgico, tras instaurar tratamiento con inhibidores de la bomba de protones, y despus de 3 meses sin otra nueva sintomatologa, comienza con estreimiento de das de evolucin, abdomen distendido, sensacin de plenitud y vmitos alimentarios incoercibles y en posos de caf. Tras realizacin de endoscopia y estudio baritado (ver imagen), cul es la causa de la clnica actual? 1. Cuadro de obstruccin por vlvulo gstrico. 2. Peritonitis por colecistitis e impactacin del cstico. 3. Pancreatitis aguda complicada. 4. lcera pptica complicada. 5. Gastritis crnica atrca. 31. Pregunta vinculada a la imagen n 17. Mujer de 47 aos con elevado consumo de tabaco y realizacin reciente de colonoscopia para biopsia de plipo colnico, acude por dolor epigstrico de aos de evolucin, asociando dispepsia y disnea inmediatamente despus de la ingesta. Se realiza una radiografa de trax en la que se observa lo que muestra la imagen. Cul crees que es la causa de su visita a Urgencias? 1. Una hernia hiatal. 2. Un neumoperitoneo. 3. Una perforacin intestinal secundaria a la colonoscopia realizada recientemente. 4. Probablemente se trate de una lcera gstrica perforada. 5. La imagen se corresponde con una situacin postprandial tpica y no patolgica. 32. Pregunta vinculada a la imagen n 17. En la paciente de la pregunta anterior, tras el diagnstico de hernia hiatal paraesofgica, qu pauta de tratamiento ser la ms adecuada? 1. IBP doble dosis para evitar, por reujo mantenido, esfago de Barret.

2. Mientras no tenga sntomas que orienten a neoplasia subyacente se trata de una patologa benigna y no es necesario tratar, slo si lo solicita el paciente. 3. Siempre se deber realizar una miotoma de Heller como tratamiento curativo. 4. Est indicada la realizacin de funduplicatura slo porque la paciente est asintomtica. 5. Funduplicatura por la clnica actual y siempre que sea una hernia tipo II, por el riesgo de efectos secundarios a este tipo de hernias. 33. Pregunta vinculada a la imagen n 18. En la analtica de un varn de 47 aos, en seguimiento en urologa por impotencia, se objetivan niveles de testosterona de 1 ng/ml (normal 3-10 ng/ml), siendo remitido a endocrinologa para estudio. En la analtica hormonal se objetiva una PRL de 1,759 ng/ml, solicitndose a continuacin una RMN (ver imagen). Dada la patologa que presenta el paciente seale lo correcto: 1. Si el paciente estuviera tomando un frmaco que produjera hiperprolactinemia, sera recomendable retirarlo y reevaluar antes de emitir el diagnstico. 2. Dado el tamao de la lesin y los niveles hormonales, lo ms probable es que el tumor produzca hiperprolactinemia por compresin. 3. Ser necesario completar el estudio del paciente con hormonas basales hiposarias, pero no ser necesario en ningn caso realizar test de estmulo para descartar dcit de ACTH. 4. Si el paciente presentara ginecomastia, se debera a la accin de la prolactina elevada sobre el tejido glandular mamario. 5. Aunque no existan dcit visuales en la anamnesis, ser necesario derivarlo a oftalmologa para realizacin de campimetra. 34. Pregunta vinculada a la imagen n 18. Respecto a la patologa del paciente anterior, seale lo falso: 1. Siempre se deben tratar aquellos tumores mayores de 1 cm, independientemente de las manifestaciones clnicas. 2. La ciruga transesfenoidal se debe emplear si persisten los defectos campimtricos, a pesar del tratamiento con agonistas dopaminrgicos a dosis mximas. 3. La radioterapia puede ser un tratamiento coadyuvante a la ciruga. 4. Recientemente se ha descrito la presencia de insuciencia valvular cardiaca en pacientes tratados con dosis altas de cabergolina, aunque su aparicin en el caso del tratamiento de la hiperprolactinemia no est clara.

-6-

EXAMEN ENARM 2/10

5. En aquellos pacientes con respuesta al tratamiento mdico es rara la recurrencia de la enfermedad. 35. Pregunta vinculada a la imagen n 19. Una mujer de 35 aos, en estudio por cefaleas de repeticin, presenta en la RMN la siguiente imagen. Como antecedentes personales de inters presenta obesidad e hipertensin arterial. El estudio hormonal hiposario se encuentra dentro de la normalidad. Segn estos datos y la imagen mostrada, seale el diagnstico ms probable: 1. 2. 3. 4. 5. 36. Craneofaringioma. Silla turca vaca. Macroadenoma hiposario no funcionante. Hipositis linfocitaria. Metstasis hiposarias.

1. Se observa una placa rosada circular. 2. Se observan reas puntiformes pigmentadas. 3. La morfologa podra ser compatible con una carcinoma basocelular supercial. 4. La morfologa podra ser compatible con un pioderma gangrenoso. 5. La morfologa podra ser compatible con una enfermedad de Bowen. 40. Pregunta vinculada a la imagen n 21. Qu tratamiento podramos realizar con el paciente? 1. 2. 3. 4. 5. 41. nicamente ciruga. nicamente radioterapia. nicamente imiquimod tpico. Ciruga o radioterapia. Ciruga o imiquimod tpico.

Pregunta vinculada a la imagen n 19. En relacin al diagnstico de la paciente, seale la opcin CORRECTA: 1. El tratamiento de eleccin ser la ciruga transesfenoidal. 2. Habitualmente se asocia a la existencia de hipopituitarismo. 3. Se debe a una destruccin linfocitaria de la hipsis. 4. La ciruga est indicada en caso de existir defectos campimtricos o fstula de lquido cefalorraqudeo. 5. Se debe iniciar tratamiento con glucocorticoides en dosis elevadas.

Pregunta vinculada a la imagen n 22. Un paciente acude a Urgencias por dolor torcico opresivo de 4 horas de evolucin. Presenta sudoracin profusa y PA de 85/40, tercer ruido izquierdo y crepitantes hmedos en el tercio basal de ambos pulmones, con una saturacin arterial de oxgeno del 80%. Se realiza un ECG (ver imagen). Cul de las siguientes opciones de tratamiento es la ms apropiada en esta situacin? 1. 2. 3. 4. 5. Lquidos intravenosos. Atropina i.v. Angioplastia primaria. Fibrinolisis con estreptokinasa. Ciruga urgente de bypass.

37.

Pregunta vinculada a la imagen n 20. Respecto a la imagen, seale la falsa: 42. 1. 2. 3. 4. 5. Se Se Se Se Se observa un intrtrigo. observan placas eritematosas. observan ppulas satlite. observa inltracin de las lesiones. observa afectacin del fondo del pliegue.

Pregunta vinculada a la imagen n 22. El diagnstico ms apropiado del paciente anterior ser: 1. Sndrome coronario agudo sin ascenso del segmento ST anterior. 2. Angina inestable. 3. Sndrome coronario agudo con ascenso del segmento ST anteroinferior. 4. Angina de Prinzmetal. 5. Infarto no Q inferior.

38.

Pregunta vinculada a la imagen n 20. El diagnstico ms probable sera: 1. 2. 3. 4. 5. Intertrigo psorisico. Eritrasma. Tia inguinal. Intertrigo candidisico. Micosis fungoide foliculotropa.

43.

Pregunta vinculada a la imagen n 22. Respecto al tratamiento de este paciente, seale la menos til de las siguientes opciones: 1. 2. 3. 4. 5. Enalapril va oral. Aspirina. Clopidogrel. Simvastatina. Tocoferol.

39.

Pregunta vinculada a la imagen n 21. Varn de 60 aos que desde hace un ao presenta una lesin asintomtica que no cura tras tratamiento con antifngicos. Cul le parece la respuesta falsa respecto a la imagen?

-7-

EXAMEN ENARM 2/10

44.

Pregunta vinculada a la imagen n 23. Un paciente fumador, sin otros antecedentes de inters, acude a Urgencias por dolor torcico de 90 minutos de evolucin. Se encuentra plido y sudoroso. La PA es de 70/40 mmHg. La auscultacin cardiopulmonar es normal. Cul de las siguientes entidades es responsable del cuadro con MENOS probabilidad a la luz del ECG (ver imagen)? 1. IAM inferior con IAM de ventrculo derecho asociado. 2. IAM inferolateral con disfuncin sistlica importante asociada. 3. IAM inferolateral con rotura cardiaca asociada. 4. IAM anteroseptal Killip IV. 5. Diseccin artica con afectacin del ostium de la coronaria derecha.

2. Ingreso, reposo en decbito lateral izquierdo y administracin de diurticos para el control de la tensin arterial. 3. Tratamiento ambulatorio con alfametildopa y seguimiento en consultas de alto riesgo semanalmente hasta la semana 40 de gestacin. 4. Ingreso para nalizar la gestacin y tratamiento antihipertensivo i.v. 5. Ingreso y tratamiento con nitroprusiato. Si no se consigue control de la tensin arterial se realizar cesrea. 48. Pregunta vinculada a la imagen n 24. En la analtica que se realiza a la paciente existe proteinuria, anemia, trombopenia, elevacin de transaminasas y elevacin de la creatinina, con pruebas de coagulacin normales. Seale la armacin FALSA respecto al cuadro que presenta la paciente: 1. El tratamiento de eleccin en esta paciente es nalizar la gestacin ya que se trata de una pre-eclampsia grave. 2. La paciente presenta una preeclampsia complicada con un sndrome HELLP. 3. Estar indicado iniciar prolaxis de la eclampsia con sulfato de magnesio. 4. El sulfato de magnesio requiere una estrecha monitorizacin por su toxicidad y el frmaco de eleccin en caso de intoxicacin es el gluconato clcico. 5. La paciente presenta una preeclampsia grave que obliga a terminar la gestacin y, dado que no ha iniciado el trabajo de parto, se realizar cesrea. 49. Los dominios N terminales de las cadenas pesadas y ligeras exhiben grandes diferencias de secuencia entre anticuerpos y reciben el nombre de regiones variables (VH y VL) que tienen las siguientes propiedades EXCEPTO una: 1. 2. 3. 4. Proporcionan especicidad frente al antgeno. Dimerizan en FV. Proporcionan especicidad de idiotipo. Contienen las porciones responsables de las propiedades efectoras. 5. No especican el isotipo. 50. Los linfocitos T cooperadores se pueden cuanticar en sangre perifrica mediante tcnicas que emplean anticuerpos monoclonales marcados con sustancias uorescentes dirigidos contra antgenos de su supercie. Cul de los siguientes fenotipos corresponde a esta subpoblacin de linfocitos? 1. CD-3+, CD-4+, CD-8+ 2. CD-3-, CD-4+, CD-83. CD-3+, CD-4+, CD-8-

45.

Pregunta vinculada a la imagen n 23. Respecto al paciente anterior, cul de estas medidas es la ms importante de cara a mejorar el pronstico a largo plazo del paciente? 1. 2. 3. 4. 5. Lquidos intravenosos. Diurticos intravenosos. Reperfusin urgente. Eplerrenona. Digitalizacin i.v.

46.

Pregunta vinculada a la imagen n 23. Cul de los siguientes marcadores de dao miocrdico es ms probable encontrar elevado en el paciente anterior en el momento de su llegada a Urgencias? 1. 2. 3. 4. 5. Mioglobina. Troponina I. Troponina T. CPK-mb. CPK-mb2.

47.

Pregunta vinculada a la imagen n 24. Paciente de 25 aos, gestante de 37 semanas, que acude a Urgencias por cefalea intensa. El embarazo ha cursado con normalidad hasta ese momento. Tensin arterial 180/120 mmHg mantenida. En la ecografa se objetiva un CIR tipo II con oligoamnios. El doppler es el que se muestra en la imagen. El registro cardiotocogrco es normal en ese momento. La conducta a seguir es: 1. Ingreso y administracin de hidralacina i.v. y sulfato de magnesio. Si se consigue control de la tensin, se continuar con tratamiento va oral con alfametildopa hasta inicio espontneo del trabajo de parto.

-8-

EXAMEN ENARM 2/10

4. CD-3-, CD-4+, CD-8+ 5. CD-3+, CD-4-, CD-8+ 51. El virus de Epstein-Barr penetra en las clulas a travs del receptor celular de complemento, es decir: 1. 2. 3. 4. 5. 52. CR2 CR1 CR3 C4BP CR4

56.

Paciente varn, de 77 aos, que viene presentando desde hace meses conductas inapropiadas, llegando incluso a robar objetos de tiendas, prdida de memoria de los acontecimientos recientes y exageracin de rasgos previos de personalidad. Cul de las siguientes pruebas complementarias NO estara indicada? 1. 2. 3. 4. 5. Niveles de B12 y folatos. Estudio tiroideo. Serologa lutica. TAC. Determinacin de cobre en sangre.

Un mdico de 39 aos acaba de terminar de leer un libro sobre tumores cerebrales y preocupado por la posibilidad de tener uno de ellos, solicita una tomografa de positrones del lbulo temporal. Cualquier signo fsico lo interpretaba como patolgico, y adjudicaba a sus cefaleas un componente orgnico. En sus aos de estudiante, enfermedad que estudiaba, enfermedad que crea tener. Cul es el diagnstico ms probable? 1. 2. 3. 4. 5. Tumor cerebral. Hipocondria. Sndrome de Mnchaussen. Simulacin. Dolor somatoforme.

57.

En relacin con la anorexia nerviosa, sealar la respuesta correcta: 1. Nunca presenta un curso progresivo hasta la muerte. 2. Normalmente comienza despus de la adolescencia. 3. Suele iniciarse con prdida de apetito. 4. La amenorrea puede preceder a la prdida apreciable de peso. 5. Rara vez consiste en un episodio nico, con posterior recuperacin del peso normal.

58.

53.

De los siguientes antipsicticos, cul tiene menos efectos sedantes? 1. 2. 3. 4. 5. Olanzapina. Tioridacina. Clorpromacina. Haloperidol. Clozapina.

Javier es un chico de 13 aos que sufre coprolalia desde hace poco tiempo. Desde los 7 aos, aproximadamente, tiene tics motores y vocales continuos. Era un nio irritable y ha tenido problemas de bajo rendimiento escolar. En relacin a la patologa que sufre este paciente, todo lo siguiente es cierto, EXCEPTO: 1. Hay un factor gentico etiolgico claro. 2. Debe debutar antes de los 15 aos. 3. En muchos casos se asocia con un trastorno por dcit de atencin e hiperactividad. 4. Debe haber mltiples tics motores para poder realizar el diagnstico. 5. El tratamiento de eleccin son los neurolpticos.

54.

Las cifras correctas de litemia en la prolaxis de trastornos bipolares es: 1. 2. 3. 4. 5. 1,5-2 mEq/l. 0,6-1,2 mEq/l. 1,2-1,6 mEq/l. 0,3-0,6 mEq/l. 2-2,3 mEq/l.

59.

Seale la FALSA respecto al terror nocturno y las pesadillas: 1. El terror nocturno suele aparecer durante el sueo profundo, por lo que es difcil despertar al sujeto. 2. Las pesadillas suceden durante la fase REM, lo que hace ms fcil despertar al paciente. 3. Las pesadillas se pueden asociar al trastorno por estrs postraumtico. 4. En el caso del terror nocturno suele existir un cortejo vegetativo, que no aparece en las pesadillas. 5. Se suelen tratar con benzodiacepinas de vida media corta como el midazolam o el triazolam.

55.

Varn de 42 aos que experimenta desde hace 2 meses un estado de intensa tristeza, prdida de inters, insomnio e ideas de culpa, muerte y suicidio. Es FALSO que: 1. La existencia de anhedonia de anticipacin, pero no de consumacin, sugiere depresin neurtica. 2. La existencia de humor ansioso e irritable descarta el diagnstico de depresin. 3. Los dcit de memoria son subjetivos y no corresponden con mediciones objetivas. 4. La existencia de insomnio de ltima fase, con despertar precoz, sugiere endogenicidad. 5. En la mayora de los casos existe disminucin del impulso sexual, apetito y peso.

60.

Un varn de 35 aos fue diagnosticado a los 18 aos de esquizofrenia paranoide, habiendo sufrido varios brotes psicticos hasta la fecha.

-9-

EXAMEN ENARM 2/10

Desde hace un tiempo la clnica del paciente se caracteriza por una gran pobreza del lenguaje, aislamiento social marcado, anhedonia y aplanamiento afectivo. Ya no presenta delirios ni alucinaciones auditivas claras. Usted escogera para el tratamiento de este paciente: 65. 1. 2. 3. 4. 5. Haloperidol. Clorpromacina. Risperidona. TEC. No existe ningn tratamiento que al menos mitigue parcialmente para los sntomas negativos de la esquizofrenia.

4. Fractura-luxacin de la extremidad proximal del hmero en un anciano de 78 aos. 5. Deformidad articular, geodas, e importante formacin osteotaria en la rodilla en un varn de 56 aos. Varn de 61 aos consulta por dolor e hinchazn en ambas orejas. Como antecedentes reere episodios de ronquera en los ltimos dos aos, as como dolor e hinchazn intermitente en algunas articulaciones, sobre todo en la rodilla y en el pie. Actualmente presenta conjuntivitis y enrojecimiento cutneo de los pabellones auriculares y del dorso de la nariz. Cul es el diagnstico ms probable? 1. Sind. de Cogan. 2. Ca. epidermoide de laringe con diseminacin sistmica. 3. Artritis reumatoide. 4. Policondritis recidivante. 5. Sind. de Reiter. 66. Un paciente de 70 aos, independiente en las actividades de la vida diaria y sin ninguna enfermedad reseable, presenta ndulos en las palmas de las manos y retraccin de los dedos en ligera exin, siendo ms notable en los dedos 4 y 5 de la mano derecha, que presentan una contractura con angulacin de 50. Cul es el tratamiento indicado en este caso? 1. Penicilamina, que interere con la sntesis del colgeno, evitando as la progresin del cuadro. 2. Actitud conservadora con vigilancia peridica, hasta que la afectacin de la mayora de los dedos sea considerable. 3. Rehabilitacin con ejercicios de estiramiento que mantengan una buena movilidad. 4. Fasciotoma en las dos manos en un primer tiempo, realizando diferidamente una fasciectoma en la derecha. 5. Fasciectoma completa, al menos en la mano derecha, pues el ndice de recurrencias es menor que con la fasciotoma. 67. Sobre la estructura del hueso, cul de las siguientes armaciones NO es cierta? 1. El hueso cortical tiene como unidad el sistema Haversiano, compuesto por un canal rodeado de lminas seas con osteocitos. 2. La funcin de los osteoclastos es la reabsorcin del hueso. 3. La matriz orgnica est compuesta principalmente por colgeno tipo II. 4. El crecimiento seo viene determinado por el platillo episario (crecimiento longitudinal) y el periostio (crecimiento concntrico). 5. El esqueleto tiene un origen mesenquimal.

61.

El uso de disulram en el tratamiento del alcoholismo es: 1. 2. 3. 4. 5. Una exposicin in vivo. Una terapia aversiva. Un refuerzo positivo. Un refuerzo negativo. Una recompensa.

62.

Para el tratamiento de un caso comprobado de encefalitis por herpes simplex, qu frmaco, de los siguientes, indicara? 1. 2. 3. 4. 5. Iododesoxiuridina. Rifampicina. Corticosteroides. Aciclovir. Vidarabina.

63.

Una mujer de cuarenta aos presenta engrosamiento en la piel de la cara, artralgias y dolor centro torcico que aumenta con la inspiracin, as como fenmeno de Raynaud; todo ello desde hace seis meses. En la radiografa de trax hay cardiomegalia y en el ecocardiograma se objetiva derrame pericrdico. Los anticuerpos anti-Scl 70 son positivos. Cul es el tratamiento de eleccin? 1. 2. 3. 4. 5. D-penicilamina. Aspirina 200 mg/da. Observacin. Prednisona. Colchicina.

64.

Cul de los siguientes casos NO es una indicacin de artroplastia de sustitucin? 1. Lesin grave de rodilla por artritis reumatoide, con dolor incontrolable con analgsicos y limitacin funcional. 2. Necrosis avascular de la cabeza femoral con colapso subcondral y aplanamiento en un paciente con LES. 3. Pinzamiento de la lnea interarticular y esclerosis subcondral en la articulacin de la rodilla en una mujer obesa de 54 aos.

-10-

EXAMEN ENARM 2/10

68.

Indique cul de las siguientes asociaciones de manifestaciones clnicas es diagnstica de lupus eritematoso sistmico: 1. Leucopenia (< 4.000 mm3, anticuerpos antiDNAds positivos, eritema malar y crisis comiciales. 2. Leucopenia (< 4.000 mm3, linfopenia (< 1.500 mm3, trombopenia (< 100.000 mm3 y anemia hemoltica. 3. Pleuritis, pericarditis, artritis y anticuerpos antinucleares. 4. Ulceras nasofarngeas, vasculitis retiniana, artritis, lupus discoide. 5. Fotosensibilidad, proteinuria, miocarditis, anticuerpos antifosfolpidos.

72.

Cul de las siguientes opciones es una caracterstica propia del sarcoma de Ewing? 1. Es un tumor maligno formador de hueso. 2. La reaccin peristica en capas de cebolla es un dato patognomnico, que da el diagnstico de una forma sencilla. 3. Clnicamente se caracteriza por una afectacin local, sin ninguna repercusin sistmica. 4. En el tratamiento deben combinarse ciruga, radio y quimioterapia. 5. Es el tumor seo primario ms frecuente en la infancia.

73.

69.

Un paciente, natural de Extremadura y pastor de cabras, reere que desde hace unos meses presenta dolor lumbar persistente y ocasionalmente irradiacin segn trayecto del citico a miembro inferior derecho. La radiografa muestra disminucin del espacio intervertebral L4-L5 y desprendimiento del ngulo episario anterosuperior. Seale su primer diagnstico: 1. Artritis de columna secundaria a TBC. 2. Salmonelosis con afectacin de columna vertebral. 3. Artritis brucelsica. 4. Osteoartrosis. 5. Osteoporosis precoz con secundario aplastamiento vertebral. Una mujer de 90 aos de edad con mal estado general y demencia senil, que sala poco a la calle y se manejaba con dicultad en sus tareas domsticas habituales, sufre una cada casual al acostarse, teniendo que ser ayudada para levantarse. Desde entonces no ha vuelto a caminar. Es diagnosticada de fractura subcapital grado III de Garden. Cul cree que sera el tratamiento ms indicado? 1. 2. 3. 4. 5. Enclavado endomedular. Reduccin y sntesis con tornillos. Traccin continua y deambulacin precoz. Prtesis parcial de cadera. Artrodesis tipo Girdlestone. 74.

Una mujer de 70 aos de edad, tras sufrir una cada, acude a su consulta por dolor e intensa inamacin a nivel de la mueca derecha. Presenta una temperatura de 37,9 C. Al realizar el estudio radiolgico se observa una imagen de doble contorno en el carpo. Qu hallazgo conrmara su sospecha diagnstica? 1. Gram y cultivo del lquido sinovial positivo. 2. Calcicaciones en los meniscos de las rodillas y en la snsis del pubis. 3. Identicacin de cristales en el lquido sinovial muy pequeos y con birrefringencia negativa. 4. Presencia en el lquido sinovial de cristales romboidales de escaso tamao con birrefringencia dbilmente positiva. 5. Un cardenal en la mueca. Un paciente varn de 62 aos, fumador, con diabetes mellitus tipo 2, presenta en las palmas de las manos bandas brosas que condiciona la retraccin de los dedos con limitacin en su extensin. Seale la respuesta ERRNEA en cuanto al proceso que afecta a este paciente: 1. La diabetes y el tabaquismo pueden haber actuado como factores predisponentes. 2. La afectacin suele ser ms intensa y precoz en el territorio cubital de la mano. 3. Es frecuente la coexistencia de bromatosis en la fascia plantar o en el pene. 4. La tendencia a la recidiva es alta, sobre todo en los casos familiares. 5. El tratamiento quirrgico se reserva para los casos progresivos con contracturas superiores a 30.

70.

71.

Paciente que 8 das despus de comenzar con penicilina por una faringitis estreptoccica presenta ebre de 38,5, urticaria, poliartralgias y linfadenopata y una prpura palpable en las extremidades inferiores. En la orina encontramos albuminuria. Usted sospechara: 1. 2. 3. 4. Prpura de Schonlein-Henoch. Reaccin de tipo enfermedad del suero. Angiitis y granulomatosis alrgica. Que lo que tena era una infeccin por el virus de Epstein-Barr. 5. Sndrome ganglionar muco-cutneo.

75.

En un paciente que ha sufrido una fractura de la disis humeral y que no puede extender la mano ni separar el pulgar, se sospechar: 1. 2. 3. 4. 5. Retraccin isqumica de Volkmann. Lesin de nervio cubital. Lesin de nervio radial. Lesin de arteria humeral. Luxacin de codo posterior.

-11-

EXAMEN ENARM 2/10

76.

Cul es el tratamiento indicado en un paciente con enfermedad de Paget localizada y poco sintomtica? 80. 1. Glucocorticoides. 2. Calcitonina. 3. Vigilancia sin tratamiento (controlando fosfatasa alcalina). 4. Etidronato. 5. Indometacina.

4. Manitol. 5. Acetazolamida. Una mujer con historia de eritema nodoso, consulta por dolor y enrojecimiento del ojo derecho. Tras la exploracin oftalmolgica es diagnosticada de uvetis anterior. La Rx de trax muestra adenopatas hiliares bilaterales. Cul es la enfermedad ms probable? 1. 2. 3. 4. 5. 81. Tuberculosis pulmonar. Infeccin postestreptoccica. Slis. Reaccin a anticonceptivos. Sarcoidosis.

77.

Varn de 35 aos homosexual VIH positivo, que 2 semanas despus de tener una uretritis comienza con artritis de tobillo derecho, 2 y 3 metatarsofalngicas derechas, dactilitis de 2 dedo del pie izquierdo y talalgia bilateral. El paciente presenta tambin erosiones indoloras superciales en glande y pstulas en las plantas de los pies. Seale la FALSA en relacin a la enfermedad que padece este paciente: 1. El agente desencadenante ms probablemente ha sido Chlamydia trachomatis. 2. Los pacientes VIH suelen tener formas especialmente agresivas. 3. Las lesiones cutneas suelen ser ms leves en los pacientes VIH que en los inmunocompetentes. 4. La zidovudina es ecaz en el tratamiento de las lesiones cutneas. 5. Se debe evitar el uso de inmunosupresores en estos pacientes.

El lser YAG se usa en la ciruga de la catarata para: 1. 2. 3. 4. 5. Operarla de forma ambulatoria. No se usa. Romper la znula durante la ciruga. Romper la znula antes de la ciruga. Romper la cpsula posterior opacicada.

82.

Una fractura oblicua completa de la pared medial de la rbita NO afectara a una de estas estructuras seas: 1. 2. 3. 4. 5. La El La La El apsis ascendente del maxilar. hueso lagrimal. lmina papircea del etmoides. apsis orbitaria interna del frontal. ala menor del esfenoides.

78.

Seale la opcin FALSA respecto a la conjuntivitis del recin nacido: 83. 1. La conjuntivitis herptica del recin nacido puede prevenirse con la cesrea, si los cultivos maternos para VHS-II son positivos. 2. La povidona yodada puede utilizarse como prolaxis para la conjuntivitis del recin nacido por gonococo y C. trachomatis. 3. En el caso de una conjuntivitis del recin nacido que aparece entre el primer y tercer da se debe sospechar Chlamydia, mientras que entre el tercer y dcimo da obliga a descartar gonococo. 4. En el caso de una conjuntivitis del recin nacido por gonococo, la clnica es hiperaguda con riesgo de perforacin, mientras que en el caso de C. trachomatis suelen existir signos de enfermedad sistmica y formacin de pannus. 5. La trasmisin tanto para gonococo como para Chlamydia es intraparto.

Todas las siguientes son posibles manifestaciones de inicio de la brosis qustica y, por tanto, indicaciones para la realizacin del test del sudor, EXCEPTO: 1. 2. 3. 4. 5. Tos crnica. Sabor salado al besarle. Plipos nasales. Esteatorrea. Invaginacin intestinal.

84.

La causa ms frecuente de hematuria macroscpica en la infancia es: 1. 2. 3. 4. 5. Sndrome de Alport. Nefropata por IgA. Hipercalciuria. Prpura de Schnlein-Henoch. Glomerulonefritis post-estreptoccica.

79.

Qu hipotensor ocular le parece el MENOS adecuado en el caso de un glaucoma agudo de ngulo estrecho? 1. Beta-bloqueante tpico. 2. Adrenrgico tpico. 3. Colinrgico tpico.

85.

Hablando de las vas sensoriales medulares, indique cul de las siguientes sensaciones se transmite por el sistema anterolateral: 1. Las sensaciones trmicas. 2. Las sensaciones tctiles nas. 3. Las sensaciones vibratorias.

-12-

EXAMEN ENARM 2/10

4. Las sensaciones de posicin. 5. Las sensaciones de presin nas. 86. Seale la armacin correcta en relacin con la infeccin por sarampin en los nios con SIDA: 1. La vacuna del sarampin est contraindicada. 2. La neumona por sarampin tiene una alta mortalidad. 3. El exantema aparece siempre y es tpico. 4. En los contactos con otros nios con sarampin no es til la gammaglobulina especca. 5. La vitamina A oral reduce las complicaciones. 87. La posicin en decbito prono durante el sueo en los lactantes se ha asociado con: 1. 2. 3. 4. 5. 88. Una mayor ganancia de peso. Mayor riesgo de neumona por aspiracin. Disminucin del trnsito intestinal. Menor incidencia de vmitos. Mayor riesgo de muerte sbita.

90.

Un nio de 10 aos se presenta en el Servicio de Urgencias con dolor inguinal y escrotal intenso, que comenz de forma brusca despus de un partido de ftbol. El nio est afectado y se observa tumefaccin y dolor en el hemiescroto derecho, no se detecta reejo cremastrico y el testculo derecho est elevado y rotado. Cul de los siguientes tratamientos considera Vd. que es el adecuado para este paciente? 1. Administracin de lquidos y antibiticos por va endovenosa. 2. Tratamiento analgsico ambulatorio. 3. Ecografa Doppler de urgencia. 4. Ingreso hospitalario para observacin clnica. 5. Exploracin quirrgica inmediata.

91.

El sndrome de Reye se acompaa de: 1. 2. 3. 4. Acidosis respiratoria. Alcalosis metablica. Hiperamoniemia. Disminucin de la creatin-fosfoquinasa plasmtica. 5. Hiperbilirrubinemia.

Para el prematuro de muy bajo peso, la leche humana puede proporcionar bajas cantidades de: 1. 2. 3. 4. 5. Caloras. Grasas. Lactosa. Calcio. Vitamina C.

92.

Un varn de 50 aos de edad tiene edemas en miembros inferiores de 2 meses de evolucin. En el sedimento de orina hay hematuria discreta y proteinuria de 4 gramos en 24 horas. Las cifras de complemento son normales. Se realiza biopsia renal y en el estudio con microscopia electrnica hay engrosamiento difuso de la membrana basal. En la glomerulonefritis que padece este enfermo, qu es caracterstico? 1. 2. 3. 4. 5. Ser ms frecuente en nios. Depsito lineal de inmunoglobulinas. Asociarse al HLA DRW3. Ser hipocomplementmica. Remisin espontnea en el 60% de los casos.

89.

Varn de 42 aos diagnosticado de miocardiopata dilatada hace 1,5 aos. Est en tratamiento con restriccin hidrosalina e IECA, pero en los dos ltimos meses relata exacerbacin de su disnea habitual hasta hacerse de reposo en los tres ltimos das. Exploracin: ingurgitacin yugular, edemas maleolares bilaterales y ligera hepatomegalia; en la auscultacin hay 3R y soplo sistlico en foco mitral; la TA es de 110/60 mmHg. La radiografa muestra inltrados perihiliares bilaterales. Hay una opcin FALSA acerca de la actitud teraputica en este paciente, selela: 1. Lo primero que se debe intentar es tratamiento mdico intensivo con oxigenoterapia, morna, diurticos y vasodilatadores. 2. Si no responde al tratamiento mdico se debe plantear el trasplante cardaco. 3. Mientras se espera el trasplante se puede colocar un baln intraartico de contrapulsacin. 4. El mecanismo del baln consiste en que se ina en sstole y se desina en distole, de manera que puede llegar a aumentar el gasto cardaco en 0,5-0,7 l/min. 5. Otra posibilidad intermedia es colocar un corazn articial.

93.

Un nio de 4 aos presenta de forma brusca dolor en la regin anterior del muslo hasta la rodilla en la pierna izquierda. Es persistente pero se acenta al caminar, y sobre todo al correr, impidindole prcticamente esta actividad. La madre relaciona este cuadro con un choque fuerte que tuvo hace unos das con otro nio cuando jugaban. Esta sintomatologa no se acompaa de malestar general ni ningn otro sntoma aparte de febrcula por las tardes. En la exploracin fsica se aprecia limitacin en la rotacin interna y la abduccin de la cadera afectada. En la analtica lo nico anormal es una leve leucocitosis y aumento de la VSG. La radiografa y la ecografa son normales. Seale la FALSA acerca del diagnstico que sospecha: 1. La evolucin es autolimitada y el tratamiento sintomtico.

-13-

EXAMEN ENARM 2/10

2. Aunque se haya resuelto el cuadro, es necesaria la restriccin de la carga y la actividad del nio durante 12 meses. 3. Hay casos espordicos que han mejorado con antihistamnicos. 4. La recidiva es frecuente en los primeros 6 meses, pero muy rara despus. 5. Es ms frecuente en el sexo masculino, en el lado izquierdo y en individuos con exceso de peso. 94. Cul de las siguientes determinaciones de laboratorio NO est indicada para la evaluacin de una posible inmunodeciencia primaria? 1. Cuanticacin de inmunoglobulinas. 2. Cultivo mixto de linfocitos. 3. Estimulacin de linfocitos con tohemaglutinina. 4. Determinacin de la CH50. 5. Valoracin de poblaciones linfocitarias. 95. En la enfermedad de membrana hialina o sndrome de distrs respiratorio neonatal existe un dcit de surfactante que se ha relacionado con todas EXCEPTO una de las siguientes situaciones: 1. 2. 3. 4. 5. 96. Prematuridad. Hijo de madre diabtica. Hipoxemia. Trastorno metablico de la sntesis de lecitina. Hijos de madres adictas a herona.

3. Intervalo. 4. Nominal no dicotmica. 5. Razn. 99. Cul de los siguientes criterios es necesario y suciente para establecer una relacin causaefecto? 1. 2. 3. 4. 5. 100. Relacin dosis-respuesta. Fuerza de la asociacin. Plausibilidad biolgica. Asociacin estadstica. Ninguno de los anteriores.

Se ha diseado un estudio de cohortes para estudiar la posible relacin entre el tabaco y la lcera pptica. Para ello se han escogido 500 personas fumadoras y 800 no fumadoras. Qu factor NO hemos tenido que considerar para elegir el tamao muestral? 1. El nivel de conanza. 2. La potencia estadstica del test de deteccin de lcera pptica. 3. El tipo de comparacin que queremos establecer (uni-bilateral). 4. El tamao de la poblacin de referencia. 5. El error tipo beta.

101.

En una determinada poblacin, el 50% son mujeres, el 68% fuma y el 40% fuma y es mujer. Si una persona seleccionada aleatoriamente es mujer, cul es la probabilidad de que sea fumadora? 1. 2. 3. 4. 5. 10% 58,8% 80% 28% 18%

Seale qu motivo de los siguientes es el ms adecuado para decidir la introduccin de un nuevo tratamiento para una determinada enfermedad, para la cual ya existe un tratamiento ecaz anterior: 1. Que el uso del nuevo tratamiento tenga una base siopatolgica mucho ms racional que el tratamiento anterior, lo que necesariamente conlleva una mayor ecacia. 2. Que, aunque no se haya demostrado an la ventaja del nuevo tratamiento en seres humanos, sta sea muy signicativa en animales mamferos superiores. 3. Que se haya comparado la ecacia del nuevo tratamiento en una serie actual de enfermos, con el tratamiento anterior en otra serie de enfermos de caractersticas comparables, de hace cinco aos, y el nuevo resulta signicativamente superior. 4. Que se haya llevado a cabo un estudio aleatorio, a ciegas, comparando ambos tratamientos y el nuevo resulte signicativamente ms ecaz. 5. Que, aunque se haya llevado a cabo un estudio aleatorio, a ciegas, comparando ambos tratamientos y la diferencia a favor del nuevo no alcance signicacin, se piense que un nuevo tratamiento siempre ser mejor al haberse descubierto ms recientemente.

97.

Cuando un nuevo tratamiento es desarrollado para prevenir la muerte, obtenindose el resultado pretendido ocurre que: 1. La prevalencia de la enfermedad puede disminuir. 2. La incidencia de la enfermedad puede aumentar. 3. La prevalencia puede aumentar. 4. Aumenta tanto la incidencia como la prevalencia. 5. No aumenta ni la incidencia ni la prevalencia.

98.

La escala de medida de una variable que contiene ms informacin es: 1. Nominal dicotmica. 2. Ordinal. 102.

La capacidad del agente infeccioso de producir una enfermedad se denomina:

-14-

EXAMEN ENARM 2/10

1. 2. 3. 4. 5. 103.

Patogenicidad. Prdromos. Contagiosidad. Infectividad. Virulencia.

108.

Cuando en los resultado de un ensayo clnico se especica que se ha realizado un anlisis por intencin de tratamiento o intencin de tratar, qu poblacin se incluye en dicho anlisis? 1. Todos los pacientes que no han abandonado el estudio por acontecimientos adversos. 2. Todos los pacientes que fueron aleatorizados. 3. Todos los pacientes que han completado el seguimiento previsto. 4. Todos los pacientes que han completado el tratamiento a estudio. 5. Todos los pacientes que no han abandonado por inecacia.

El nmero de extracciones que hay que repetir en un centro de salud se mantiene estable durante los ltimos 12 meses, con una media del 12%. Qu debe hacerse para conseguir una media inferior al 5%? 1. Cambiar el proceso. 2. Motivar a las personas que intervienen en el proceso. 3. Identicar a las personas responsables de los fallos. 4. No puede considerarse estable solo con 12 mediciones. 5. Realizar un estudio de evaluacin.

109.

Cuando identicamos la proporcin de pacientes con enfermedad, en los que el resultado de la prueba problema es negativa, hacemos referencia a: 1. 2. 3. 4. 5. Tasa de falsos positivos. Tasa de falsos negativos. Sensibilidad. Especicidad. Valor predictivo positivo.

104.

Le encargan un estudio sobre la prevalencia de hepatopata alcohlica en su comunidad autnoma. Vd. determinara este parmetro mediante: 1. 2. 3. 4. 5. Estudio de cohortes. Estudio de casos y controles. Estudio de corte o transversal. Ensayo clnico cruzado. Anlisis de la varianza.

110.

Respecto a los parmetros que miden la validez interna de un test, seale la opcin INCORRECTA: 1. La sensibilidad se puede denir como la capacidad de un test para clasicar a los enfermos como enfermos. 2. La especicidad se puede denir como la capacidad de un test para clasicar a los sanos como tales. 3. Si la especicidad de un test es del 95%, signica que de cada 100 individuos sanos, este test es capaz de detectar 95. Los 5 que no detecta, que tambin estn sanos y que el test identica como enfermos, son falsos positivos. 4. Al disminuir la sensibilidad de un test, disminuyen los falsos positivos. 5. Al aumentar la especicidad de un test, disminuyen los falsos negativos.

105.

Cul de los siguientes NO puede ser el resultado de una prueba estadstica de contraste de hiptesis? 1. 2. 3. 4. 5. Aceptacin de la hiptesis nula. Rechazo de la hiptesis nula. Aceptacin de la hiptesis alternativa. No rechazo de la hiptesis nula. Rechazo de la hiptesis alternativa.

106.

A la capacidad que tiene un test estadstico para detectar diferencias signicativas se le denomina: 1. 2. 3. 4. 5. Precisin. Ajuste. Sesgo. Potencia. Signicacin.

111.

Se entiende por error tipo confounding: 1. Error producido por la baja de frecuencia de un proceso. 2. Error producido por la eleccin de una muestra no representativa de la poblacin. 3. Error producido por la accin de un factor no incluido en el estudio, pero que inuye sobre las variables estudiadas. 4. Error producido por la utilizacin de una escala de medicin inadecuada. 5. Error producido por la seleccin inadecuada de los controles.

107.

Los trminos que intervienen en la obtencin de la presin de urgencias son: 1. El nmero de camas ocupadas y los ingresos urgentes. 2. Los ingresos programados y los ingresos urgentes. 3. Los ingresos urgentes desde consulta y los ingresos urgentes hospitalarios. 4. Los ingresos urgentes y los ingresos totales. 5. Los ingresos programados y el nmero de camas de urgencias.

112.

Se realiza un estudio para calcular la incidencia de hepatitis B en 50 varones homosexuales HB-

-15-

EXAMEN ENARM 2/10

sAg y Ac anti-HBs negativos. La duracin del estudio es de 10 aos, trascurridos los cuales tenemos que hay 5 casos de HBsAg+. Suponiendo que no existe el perodo ventana y que todos ellos se infectaron durante el 8 ao de seguimiento, calcula la IA y la DI (respectivamente): 1. 2. 3. 4. 5. 113. 0,01 y 0,2 0,2 y 0,02 0,1 y 0,03 0,3 y 0,1 0,1 y 0,01

2. Nunca ocurren prdidas de individuos en el seguimiento. 3. No permiten el clculo de la razn de incidencias. 4. No es adecuado para la formulacin de nuevas hiptesis de causalidad. 5. Son los estudios de eleccin en enfermedades de perodo de induccin largo. 117. El grado en que una medicin representa el verdadero valor del fenmeno que se desea medir se conoce como: 1. 2. 3. 4. 5. 118. Validez. Precisin. Sensibilidad. Repetibilidad. Globalidad.

Respecto al anlisis de regresin y correlacin, seale lo FALSO: 1. Estudian la relacin entre dos variables cualitativas. 2. En la regresin interviene una variable dependiente y una o ms independientes. 3. El coeciente de correlacin es una medida de asociacin lineal entre dos variables cuantitativas. 4. La pendiente de la recta de la regresin lineal indica lo que aumenta la variable dependiente por unidad de aumento de la independiente. 5. La ordenada en el origen indica el valor de la variable dependiente cuando la independiente vale 0.

Si se establece que en un centro de salud el tiempo de espera para una extraccin de sangre nunca superar las 72 horas, se est construyendo un criterio: 1. 2. 3. 4. 5. Implcito. Implcito emprico. Implcito normativo. Explcito emprico. Explcito normativo.

119. 114. Se comparan los resultados acadmicos (aprobado/suspenso) de dos colegios, uno privado y el otro pblico. Qu test sera el ms apropiado? 1. 2. 3. 4. 5. 115. ANOVA. Chi-cuadrado. Regresin lineal. t de Student. U de Mann-Whitney. 120.

Sealar la FALSA referente a la odds ratio: 1. Mide la fuerza de la asociacin. 2. No tiene unidades. 3. Es una medida de asociacin utilizada en los estudios casos y controles. 4. Puede identicar factores de proteccin. 5. Es un buen estimador del riesgo relativo slo cuando estudiamos una enfermedad muy frecuente. Varn de 9 aos de edad que acude a Urgencias por presentar un cuadro de dolor abdominal que ha comenzado en epigastrio, para posteriormente localizarse en la fosa ilaca izquierda, de 3 horas de evolucin. A la exploracin el paciente presenta una temperatura de 38,8 C. El abdomen es doloroso de forma difusa, ms localizado en la fosa ilaca izquierda, con importantes signos de irritacin peritoneal. Ante tales datos deberemos pensar que el paciente puede presentar los siguientes diagnsticos EXCEPTO: 1. 2. 3. 4. 5. Apendicitis aguda. Diverticulitis de Meckel. Neumona basal izquierda. Invaginacin intestinal. Salpingitis aguda.

En una ciudad de Estados Unidos se realiz un estudio para tratar de conocer la posible asociacin entre la obesidad y la aparicin de HTA durante el embarazo. Para ello se siguieron a 100 mujeres obesas y 200 mujeres no obesas durante el embarazo. Aparecieron 60 casos de eclampsia, de los cuales 40 correspondieron a mujeres obesas. Cul es el riesgo relativo (RR)? 1. 2. 3. 4. 5. 4 6 5 0,66 No se puede calcular el riesgo relativo.

116.

Seale la correcta en relacin con los estudios de cohortes: 121. 1. Los estudios de cohortes estn indicados en enfermedades raras.

Cul de las siguientes medidas le parece prioritaria ante un paciente en shock hemodinmico tras accidente de coche con un traumatismo

-16-

EXAMEN ENARM 2/10

traco-abdomino-plvico con herida abierta en hemitrax izquierdo? 1. 2. 3. 4. 5. 122. Aspiracin de los vmitos. Aporte de volumen. Laparotoma exploratoria urgente. Administracin de hemoderivados. Rx trax. 125.

5. Carcinoma de ngulo colo-esplnico. Cecostoma de descarga. Cul de las siguientes complicaciones no es frecuente en los pacientes que han recibido un trasplante heptico alognico? 1. 2. 3. 4. 5. 126. Trombosis de la vena porta. Rechazo hiperagudo. Trombosis de la arteria heptica. Infecciones por Citomegalovirus. Enfermedad linfoproliferativa.

Un paciente de 65 aos, sin antecedentes de inters, ha sido operado hace 3 das de un melanoma localizado en la pierna izquierda, habindosele realizado extirpacin de la lesin tumoral con mrgenes adecuados, tanto en supercie como en profundidad, y linfadenectoma inguinal izquierda. Actualmente presenta tumefaccin indolora de la herida inguinal, sin signos inamatorios. Seale cul, de entre los siguientes, le parece el diagnstico ms probable: 1. 2. 3. 4. 5. Hematoma. Hernia inguinal postoperatoria. Infeccin de la herida. Seroma infectado. Seroma.

Una paciente de 65 aos consulta por astenia de cuatro meses de evolucin. La exploracin es normal, incluyendo un tacto rectal. En los datos analticos nicamente destaca una hemoglobina de 9,5 g/dl, hematocrito 29% y VCM 72 . La ferritina es de 8 ng/ml. Una esofagogastroduodenoscopia fue normal. Qu prueba le hara a continuacin? 1. 2. 3. 4. 5. TAC abdominal. Ecografa abdominal. Electroforesis de hemoglobinas. Colonoscopia. Trnsito gastrointestinal con bario.

123.

Un paciente de 80 aos con EPOC ingresa con ictericia obstructiva y ebre alta. En el estudio ecogrco presenta colelitiasis y coledocolitiasis que se conrma por colangiopancreatografa retrgrada. Cul es el tratamiento de eleccin? 1. 2. 3. 4. 5. Ciruga biliar. Papilotoma endoscpica. Acido quenodesoxiclico oral. Antibiticos de amplio espectro. Colecistostoma percutnea o quirrgica.

127.

Una paciente de 34 aos ha sido diagnosticada de enfermedad celaca con histologa y serologa, aunque cumple el tratamiento estrictamente no ha mejorado tras tres meses, e incluso ha perdido 7 kilos de peso y tiene febrcula. En qu pensara usted en primer lugar? 1. El diagnstico de celaca no es cierto. 2. Realmente no hace bien la dieta. 3. Buscara otra enfermedad asociada como un hipotiroidismo. 4. Tres meses es poco tiempo. 5. Descartara un linfoma intestinal.

124.

Un enfermo de 60 aos con antecedentes de trastornos del hbito intestinal en los ltimos tres meses, ingresa por distensin abdominal, sensacin nauseosa y vmitos de carcter entrico. La radiologa abdominal (simple, bipedestacin) demuestra abundantes imgenes hidroareas en arcos de bveda y tubos de rgano con extrema dilatacin (haustras) en colon derecho y transverso, sin evidencia de colon descendente ni recto. Cul de los propuestos ser el diagnstico y en caso de conrmarse, el tratamiento ms correcto? 1. Perforacin de divertculo sigmoideo con absceso paraclico. Reseccin sigmoidea y colostoma. 2. Obstruccin intestinal por bridas. Laparotoma y liberacin del segmento afectado. 3. Neoplasia de ngulo colo-heptico. Reseccin segmentaria y anastomosis coloclica. 4. Carcinoma de ngulo colo-esplnico. Colectoma subtotal y anastomosis ileoclica (sobre colon descendente).

128.

Un paciente de 50 aos est realizando un tratamiento con 300 mg diarios de ranitidina por una lcera gstrica. A las cuatro semanas de iniciado el tratamiento se le realiza una endoscopia en la que se observa que el tamao de la lcera se ha reducido mnimamente. Cul debe ser la actitud teraputica a partir de este momento? 1. Continuar el tratamiento, ya que, aunque lentamente, la lcera responde al mismo. 2. Realizar una nueva endoscopia a las 8 semanas, y si la cicatrizacin no es completa realizar tratamiento quirrgico. 3. Aumentar la dosis de ranitidina o aadir omeprazol y tomar biopsias de la lesin. 4. Tomar biopsias, si no se encuentran signos de malignidad continuar tratamiento hasta la 8 semana, en que se repetir la endoscopia. 5. Repetir endoscopia a las 8 semanas, si la lcera ha cicatrizado descartamos con seguridad su malignidad.

-17-

EXAMEN ENARM 2/10

129.

Mujer de 40 aos, consulta por diarrea crnica acuosa uctuante, sin esteatorrea ni sangre. La colonoscopia es macroscpicamente normal y en la biopsia se objetiva inltrado por linfocitos y clulas plasmticas en la lmina propia e intraepitelial. Respecto a la enfermedad que probablemente presenta esta paciente seale la FALSA: 1. La etiologa es desconocida aunque se han descritos casos asociados a frmacos y a enfermedades autoinmunes. 2. En la evolucin de la enfermedad en esta paciente podra observarse engrosamiento de la banda subepitelial de colgeno. 3. Tiene un curso crnico recidivante y se han descrito casos de remisin espontnea. 4. Debe instaurarse tratamiento con dieta sin gluten. 5. El pronstico en general es bueno y no hay evidencia de predisposicin a cncer)

pncreas. Al explorar la cavidad qustica aparece una lesin sospechosa que se biopsia, revelando cistoadenocarcinoma. No se observaron ganglios linfticos. Cul, de los enumerados, es el tratamiento ms apropiado? 1. 2. 3. 4. 5. Drenaje externo. Quistoyeyunostoma en Y de Roux. Escisin radical. Biopsia solamente. Canulacin de la arteria heptica para infusin de quimioterapia.

133.

Ante un varn de 18 aos de edad diagnosticado previamente de esquizofrenia paranoide y que ingresa en Urgencias tras ingerir aguafuerte, NUNCA estar indicado una de las siguientes maniobras: 1. 2. 3. 4. 5. Induccin del vmito. Ingreso hospitalario. Analgesia. Endoscopia alta. Sedacin del paciente.

130.

Un enfermo cirrtico, ingresado por una hemorragia digestiva alta por varices ya tratada con medidas endoscpicas, ha comenzado a tener valores de creatinina en sangre elevados. Indique cul de las armaciones le parece INCORRECTA: 1. Si no se ha repuesto la volemia de manera suciente es lgico que presente insuciencia renal. 2. Sera til determinar la natriuresis, ya que si sta est disminuida la insuciencia renal puede ser de origen prerrenal. 3. Habra que revisar el tratamiento del enfermo, sobre todo diurticos y antibiticos. 4. La determinacin del sodio en sangre es til para el diagnstico. 5. Solicitaremos un sedimento urinario, ya que si ste es patolgico hemos de pensar en un sndrome hepatorrenal.

134.

Indique lo correcto respecto al colangiocarcinoma: 1. Se asocia a infeccin crnica por virus de hepatitis B. 2. Produce dolor severo. 3. Cursa con elevacin de la alfa-fetoprotena. 4. Se asocia a colestasis crnica. 5. Tiene gran tendencia a metastatizar.

135.

131.

En relacin con la hepatitis crnica por el virus B, indique cul de las siguientes le parece cierta: 1. La presencia de HBeAg y ADN del virus B en sangre siempre implica hepatitis crnica activa. 2. La determinacin de HBcAg en sangre implica replicacin. 3. Ante una infeccin por el virus B conrmada basta con conocer la histologa heptica, pudiendo prescindir del estudio de replicacin. 4. El 10 al 15% de las formas replicativas se hacen no replicativas al cabo de un ao de forma espontnea. 5. La deteccin de ADN viral integrado en el genoma del hepatocito supone mayor infectividad.

Un varn de 75 aos es ingresado porque desde el da anterior sufre hemorragias rectales y se encuentra mareado. Hace cuatro aos se le resec un aneurisma de aorta abdominal y se coloc un injerto. Hace diez aos sufri un infarto anterior y ha tenido episodios ocasionales de angina de esfuerzo. El da del ingreso ha tenido cinco episodios de angina de reposo que han desaparecido con nitroglicerina. El hematocrito es de 18%. Cul de los siguientes trastornos le parece MENOS probable? 1. 2. 3. 4. 5. Ulcera duodenal. Hemorragia diverticular. Fstula aortoduodenal. Angiodisplasia de colon. Adenoma velloso del recto.

136.

Cul es la base gentica molecular de la mayora de sndromes de cncer familiar? 1. Mutacin dominante en lnea germinal de un proto-oncogen. 2. Mutacin no dominante en lnea germinal de un proto-ongogen y mutacin dominante somtica del segundo alelo.

132.

Un hombre de 50 aos con pancreatitis alcohlica es intervenido quirrgicamente para tratamiento de pseudoquiste a nivel de cola de

-18-

EXAMEN ENARM 2/10

3. Mutacin en lnea germinal de un alelo de un gen supresor de tumores e inactivacin somtica del segundo alelo. 4. Mutacin en lnea germinal de los dos alelos de un gen supresor de tumores. 5. Herencia de tipo polignica. 143. 137. La neoplasia ms frecuente en los inmunodeprimidos es: 1. 2. 3. 4. 5. 138. Linfoma de Hodgkin. Linfoma no Hodgkin. Timoma. Carcinoma de pulmn. Carcinoma de estmago.

1. 2. 3. 4. 5.

Desensibilizacin sistemtica. Implosin e inundacin. Terapia aversiva. Entrenamiento asertivo. Psicoterapia breve y focal.

Cul de las siguientes armaciones es cierta respecto al tratamiento con lovastatina? 1. Acta inhibiendo la absorcin de las sales biliares. 2. Inhibe la HMG-CoA reductasa, que es la enzima limitante en la sntesis de los triglicridos endgenos. 3. No tiene efectos secundarios a nivel heptico. 4. Su utilizacin junto con genbrocil previene la aparicin de miopata. 5. Disminuye la produccin de LDL y aumenta la depuracin heptica de LDL al aumentar el nmero de receptores.

Pensando en la teratogenia de estos frmacos utilizados en dermatologa, cul de estos perodos de anticoncepcin le parece INCORRECTO? 1. 2. 3. 4. 5. Metotrexato = 12 semanas. Etretinato = 2 aos. Acitretina = 48 meses. Ciclosporina = mnimo durante el tratamiento. Isotretinona = 1 - 2 meses.

144.

Seale el mayor factor de riesgo relativo para desarrollar melanoma maligno: 1. Nmero total de nevus melanocticos congnitos. 2. Fotoexposicin acumulada e intermitente. 3. Nevus atpicos mltiples. 4. Historia personal de melasma. 5. Fototipos cutneos I y II.

139.

El concepto de biodisponibilidad de un frmaco expresa: 1. La cantidad de principio activo administrado en una dosis nica. 2. La cantidad total de principio activo administrado en 24 horas. 3. La cantidad total de frmaco disponible para ser eliminado en 24 horas. 4. La cantidad de frmaco que llega a la circulacin sistmica. 5. La forma farmacutica del medicamento (tabletas, polvo, inyectable, etc.).

145.

De los medicamentos que se relacionan a continuacin, seale cul tiene ms riesgo de provocar disforia, cuadros psicomotores y alucinaciones: 1. 2. 3. 4. 5. Naloxona. Morna. Pentazocina. Buprenorna. Meperidina.

140.

Son efectos secundarios del tratamiento con captopril, EXCEPTO: 146. 1. 2. 3. 4. Tos. GN membranosa. Hiperpotasemia. Estimulacin de la produccin de eritropoyetina. 5. Angioedema.

Con respecto a la epidemiologa del VIH, una es FALSA: 1. Uno de cada 4 casos de VIH en Europa, aparece en Espaa. 2. El SIDA es la primera causa de muerte en algunas zonas entre los 25 y los 35 aos. 3. El grupo de riesgo ms frecuente en Espaa son los homosexuales. 4. La causa ms frecuente de morbi/mortalidad en Espaa de SIDA es la tuberculosis. 5. La transmisin heterosexual est en aumento en Espaa.

141.

Cul es el mecanismo de accin del metrotrexato? 1. 2. 3. 4. 5. Inhibe Inhibe Inhibe Inhibe Inhibe la la la la la enzima dihidrofalato reductasa. enzima ribonucletido reductasa. dihidropteroato sintetasa. monoaminooxidasa. DNA polimerasa.

147.

142.

Cul de las siguientes formas de psicoterapia NO est dentro de las terapias de modicacin de conducta?

Un joven de 18 aos, con antecedentes de alergia a plenes y eccema, acude a Urgencias por dicultad respiratoria progresiva, con broncoespasmo muy severo que obliga a actuacin teraputica inmediata. Seale de los siguientes datos observados, cul es el

-19-

EXAMEN ENARM 2/10

que MENOS nos indica la gravedad de la situacin? 1. 2. 3. 4. 5. 148. Hiperinsuacin torcica. Sibilancias. Hipercapnia. Pulso paradjico. Uso de msculos accesorios respiratorios.

peso 7 kg en 6 semanas. El recuento de clulas blancas sanguneas es de 13.500 mm3. Tiene antecedentes de asma leve. Una radiografa de trax muestra inltrados pulmonares perifricos. El diagnstico ms probable sera: 1. 2. 3. 4. 5. 153. Fibrosis pulmonar idioptica. Proteinosis alveolar. Polimiositis. Neumona eosinla crnica. Linfangioleiomiomatosis.

Cul es el sntoma clnico predominante en las brosis pulmonares? 1. 2. 3. 4. 5. Expectoracin. Hemoptisis. Disnea de esfuerzo. Dolor costal. Crisis de disnea.

149.

Paciente de 20 aos que jugando al baloncesto ha tenido un sncope. La nica alteracin encontrada en las pruebas realizadas es la presencia de un bloqueo AV completo a su llegada a Urgencias y adenopatas hiliares bilaterales en la Rx de trax. El diagnstico ms probable ser: 1. 2. 3. 4. 5. Lupus eritematoso sistmico. Tuberculosis. Carcinoide. Sarcoidosis. Asbestosis.

Una mujer de 38 aos reere disnea progresiva desde hace unos 2 aos. La exploracin fsica revela incremento de la presin venosa yugular y reduccin del pulso carotdeo, as como aumento del componente pulmonar del segundo tono cardaco y del tercero y cuarto tonos derechos. Rx trax: agrandamiento de las arterias pulmonares. ECG: desviacin del eje a la derecha con signos de hipertroa ventricular derecha. Otros datos: hipoxemia, hipocapnia, DLCO reducida. Sin realizar otros estudios para completar el diagnstico, usted dira que previsiblemente esta enferma requiere para su tratamiento: 1. 2. 3. 4. 5. Ejercicio fsico diario. Vasodilatadores. Digitlicos a altas dosis. Antibrinolticos. Antiparasitarios frente a anquilostoma.

150.

Paciente politrasfundido por anemia que no responda a tratamiento con hierro, acude con taquipnea y disnea, junto a malestar general. En la gasometra se aprecia hipoxemia, y la radiografa torcica ofrece extensos inltrados alveolointersticiales difusos. No se consigue mejorar su hipoxemia a pesar del tratamiento con oxigenoterapia (FiO2>50%). Su diagnstico ser: 1. 2. 3. 4. 5. Sndrome de distrs respiratorio agudo. Enfermedad injerto contra husped. Primoinfeccin por VIH. Hemorragia alveolar difusa. Neumona por CMV.

154.

Seale, de la siguiente lista, la causa menos frecuente de ndulo pulmonar benigno: 1. 2. 3. 4. 5. Lipoma. Granuloma. Hamartoma. Teratoma. Fstula arteriovenosa.

155.

151.

El mediastino puede ser dividido en tres compartimentos (anterosuperior, medio o visceral y posterior). De los tumores que se enumeran a continuacin, seale cul NO suele localizarse en el mediastino anterosuperior: 1. 2. 3. 4. 5. Fibrosarcoma. Ganglioneuroma. Hemangioma. Linfangioma. Linfoma.

Un paciente presenta hipercapnia crnica, hipoxemia, disminucin de la presin inspiratoria y espiratoria mxima (PI mx y PE mx) y reduccin de las respuestas electromiogrcas del diafragma. Cul de las siguientes trastornos presenta con ms probabilidad? 1. 2. 3. 4. 5. Sndrome de hipoventilacin alveolar primaria. Miastenia gravis. Toracoplastia. Hemorragia del tronco enceflico. Poliomielitis bulbar.

156.

152.

Un hombre de 54 aos tiene una tos no productiva y disnea de esfuerzo. Tambin nota febrcula, malestar general y una prdida de

Acude a urgencias un varn de 58 aos, con tinte ciantico, abotargamiento facial y quemosis conjuntival. En la analtica existe una clara cianosis (ms de un 5% de hemoglobina reducida), y en la exploracin el paciente est taquipnico y sobre el trax existen varios cordones venosos. Usted sospecha:

-20-

EXAMEN ENARM 2/10

1. 2. 3. 4. 5. 157.

Carcinoma pulmonar tipo oat-cell. Linfoma mediastnico. Insuciencia cardaca dercha. Tromboembolismo pulmonar. Carcinoma pulmonar tipo adenocarcinoma.

Varn de 65 aos, diagnosticado hace 25 aos de estenosis mitral de origen reumtico, acude a consulta por presentar un empeoramiento de su disnea habitual de medianos esfuerzos. Presenta en la placa de trax lneas B de Kerley y un derrame pleural bilateral. Respecto a este paciente, cul de estas armaciones es FALSA? 1. La causa subyacente del derrame pleural es probablemente la insuciencia cardiaca secundaria a la estenosis mitral que padece este enfermo. 2. El mejor tratamiento del derrame pleural de este paciente lo constituyen los diurticos. 3. Debe hacerse una toracocentesis diagnstica previa al tratamiento para descartar otras posibles causas, ya que el tratamiento diurtico modicar las caractersticas bioqumicas del lquido pleural en unos das. 4. En la toracocentesis se encontrara probablemente un cociente protenas en lquido pleural/ protenas sricas < 0,5. 5. Si en la toracocentesis se encontrase una LDH en lquido pleural superior a los dos tercios del lmite superior se la normalidad para suero, habra que dudar del diagnstico de derrame pleural secundario a insuciencia cardiaca.

1. La ms frecuente es el adenocarcinoma. 2. El adenocarcinoma y el anaplsico de clulas grandes son perifricos. 3. La probabilidad de cncer pulmonar en ex fumadores no se aproxima a los no fumadores hasta 15 aos. 4. Los tumores perifricos son clnicamente ms silenciosos que los centrales. 5. El sndrome de la vena cava superior orienta el diagnstico a carcinoma anaplsico de clulas grandes. 161. Cul es el germen ms frecuente en la infeccin urinaria en la infancia? 1. 2. 3. 4. 5. 162. Proteus mirabilis. Pseudomona aeruginosa. Haemophilus inuenzae. Escherichia coli. Staphilococcus saprophiticus.

Varn de 52 aos que consulta por cuadro de 2 meses de evolucin consistente en astenia, febrcula vespertina, prdida de 5 kg de peso y tos seca, que en los dos ltimos das se ha acompaado de esputos hemoptoicos. No es fumador y trabaja en el campo. Aporta radiografa de trax realizada hace 24 horas con patrn destructivo biapical. La exploracin fsica es normal. Qu prueba recomendara en este momento? 1. Auramina de esputo. 2. Anticuerpos monoclonales para Pneumocystis carinii en esputo. 3. Fibrobroncoscopia. 4. Puncin transtraqueal. 5. Inmunouorescencia directa para Legionella pneumophila en esputo.

158.

Paciente con EPOC tipo bronquitis crnica al que se hace una gasometra arterial y presenta: pH = 7,29; PaCO2 = 62 mmHg, PaO2 = 50 mmHg. La causa ms frecuente de esta descompensacin sera: 163. 1. 2. 3. 4. 5. Neumona por H. inuenzae. Infeccin del rbol traqueobronquial. Neumona por neumococo. Tromboembolismo pulmonar. Neumotrax.

Ante el hallazgo de un adenocarcinoma renal limitado a la cpsula de Gerotta, con trombosis tumoral de la vena renal pero sin afectacin histolgica de ganglios linfticos, el estadio tumoral segn la clasicacin TNM corresponde a: 1. 2. 3. 4. 5. T1a T2 T3b T3c T4

159.

Varn de 57 aos con carcinoma metastsico de prstata. Sufre episodio de disnea brusca y dolor torcico y se realiza el diagnstico de tromboembolismo pulmonar. Cunto tiempo mantendra la anticoagulacin en este paciente? 164. 1. 2. 3. 4. 5. Cinco das. Tres meses. Seis meses. Un ao. De por vida.

Dentro de las anomalas congnitas ureterales que se citan a continuacin, la que presenta una mayor incidencia es: 1. 2. 3. 4. 5. Urter retrocavo. Megaurter. Ureterocele. Urter retroilaco. Duplicidad ureteral.

160.

Respecto a las neoplasias pulmonares malignas seale la FALSA:

-21-

EXAMEN ENARM 2/10

165.

Un varn de 50 aos de edad tiene unas cifras de glucemia elevadas desde hace diez aos, con mal control. Las cifras de creatinina srica estn elevadas y el aclaramiento de creatinina es de 30 ml/min. La proteinuria es de 2,5 g/da. Qu tratamiento est CONTRAINDICADO en este paciente? 1. 2. 3. 4. 5. Administracin Administracin Administracin Administracin Administracin de de de de de insulina. antidiabticos orales. bicarbonato. hipotensores. diurticos.

169.

Un varn de 45 aos, fumador, acude a su consulta por sndrome miccional, PSA elevado, tacto rectal sospechoso y linfedema escrotal. El resultado de la biopsia prosttica mediante eco transrectal y biopsia demuestra un adenocarcinoma de prstata Gleason de 9. En este sentido es cierto que: 1. Tiene ms del 70% de probabilidad de tener afectacin linftica. 2. La enfermedad est, probablemente, localizada. 3. La biopsia transperineal tomar muestras ms precisas que la transrectal. 4. Tendr mayor probabilidad de afectacin heptica y pulmonar. 5. Es imposible una diseminacin hematgena.

166.

Paciente con historia de clicos nefrticos de repeticin expulsivos, cuyo estudio metablico muestra una hipercalciuria idioptica no influenciada por el ayuno ni la ingesta. De los siguientes, cul es el tratamiento adecuado? 1. 2. 3. 4. 5. Fosfato de celulosa. Dieta hipoproteica y ortofosfatos. Alopurinol. Ingesta abundante de lquidos y tiacidas. Alcalinizacin de la orina y D-penicilamina.

170.

Cul de los siguientes datos es el ms caracterstico de una nefritis intersticial aguda? 1. Eosinolia en sedimento urinario. 2. Eosinolia perifrica y aumento de los niveles de IgG en suero. 3. Fiebre asociada a I.R.A. 4. Infeccin por estreptococos. 5. Exantema.

167.

Un varn de 40 aos de edad tiene hematuria y proteinuria en rango nefrtico. Las cifras de complemento son normales y la cifra de creatinina srica es de 2 mg/dl. Se practica una biopsia renal, encontrndose con microscopia electrnica una membrana basal en capa de hojaldre. El enfermo padece, con toda probabilidad: 1. 2. 3. 4. 5. Enfermedad de Alport. Nefronoptisis. Rin en esponja. Poliquistosis renal. Pielonefritis aguda.

171.

Mujer de 30 aos ingresa en urgencias por ebre alta y taquipnea, adems de un cierto grado de deshidratacin. En la analtica de urgencia se observan los siguientes datos: urea de 65 mg/dl, Na+: 150 mEq/dl ,Cl-: 84 mEq/ dl, K+:6,1 mEq/dl y bicarbonato: 15 mEq/dl. En orina hay oliguria y glucosuria. Es cierto que: 1. Estos datos son compatibles con acidosis tubular renal. 2. Existe aumento del anin gap. 3. La pCO2 ser probablemente > de 45 mmHg. 4. Tiene acidosis respiratoria. 5. Tiene una alcalosis metablica.

168.

La acidosis tubular renal (ATR) proximal o tipo II, consisite en un defecto en la reabsorcin proximal del bicarbonato ltrado, lo que causa un aumento del bicarbonato que llega a la nefrona distal y nalmente a la orina (bicarbonaturia). En relacin con este proceso, seale la armacin FALSA: 1. La bicarbonaturia est presente cualquiera que se la concentracin srica de CO3H-. 2. La bicarbonaturia disminuye a medida que los hace la concentracin srica de CO3H-. 3. La bicarbonaturia desaparece cuando la concentracin srica de CO3H- es inferior a 16 mmol/L. 4. La bicarbonaturia aumenta cuando se normaliza la concentracin srica de CO3H- con la administracin de CO3HNa. 5. La bicarbonaturia se acompaa de aumento de la excrecin urinaria de potasio.

172.

Para medir el ujo plasmtico renal se puede utilizar el aclaramiento de: 1. 2. 3. 4. 5. Inulina. Acido paraminohiprico. Acido fosfrico. Vitamina B12. Iodotalamato marcado con I125.

173.

La creatinina en sangre no se suele elevar por encima de los valores normales hasta que el ltrado glomerular no es inferior a: 1. 2. 3. 4. 5. 100 ml/minuto. 50 ml/minuto. 25 ml/minuto. 10 ml/minuto. Estadios terminales.

-22-

EXAMEN ENARM 2/10

174.

Mujer de 27 aos que consulta por amenorrea de 3 meses. Un embarazo y parto normales hace 5 aos. Ex usuaria de drogas por va inhalatoria, negando el uso de la va parenteral. Su pareja sexual es ADVP activo y VIH positivo en estadio C3. En la serologa del primer trimestre se diagnostican anticuerpos anti-VIH positivos en la gestante. Debemos informarle de todo lo siguiente, EXCEPTO: 1. La transmisin vertical o materno-fetal de la infeccin por VIH-1 constituye el principal modo de adquisicin de la infeccin en los nios. 2. La introduccin de la zidovudina en monoterapia para disminuir la transmisin vertical del VIH (protocolo 076) supuso una disminucin del 20-30% al 8%. 3. La existencia de otras ETS asociadas, pueden incrementar el riesgo de transmisin vertical. 4. Deber renunciar a la lactancia materna, puesto que esta duplica el riesgo de transmisin madre-hijo. 5. Est aceptada la cesrea electiva de forma sistemtica como nica va del parto.

biamnitica en la semana 36+4 cuyo curso ha sido normal; la exploracin revela un crvix borrado, con 5 cm de dilatacin, blando y centrado; primer gemelo en presentacin ceflica, segundo en transversa, bolsa ntegra y amnioscopia negativa. Cul es la actitud correcta de cara al parto? 1. Realizar una cesrea urgente, ya que se trata de una gestacin biamnitica. 2. Dejar evolucionar el parto por va vaginal. 3. Dejar evolucionar el parto vaginal del primer gemelo y realizar una cesrea no electiva del segundo. 4. Iniciar la administracin de tocolticos, puesto que se trata de una gestacin de 36 semanas y es previsible que los fetos sean muy inmaduros. 5. Realizar una cesrea urgente puesto que el segundo gemelo est en transversa. 178. Cul es la lesin histolgica caracterstica de la nefropata gravdica? 1. 2. 3. 4. 5. 179. Depsitos densos en membrana basal. Necrosis tubular. Proliferacin extracapilar. Endoteliosis glomerular. Proliferacin endocapilar.

175.

Qu es correcto respecto a la funcin endocrina de la placenta? 1. Los valores de gonadotropina corinica humana (HCG) son mximos a las 30-35 semanas de gestacin. 2. Los valores de beta-HCG inferiores a la media pueden ser marcador bioqumico de cromosomopata. 3. El lactgeno placentario tiene estructura similar a las hormonas LH, FSH y TSH, compartiendo con ellas la subunidad alfa. 4. El lactgeno placentario permite mayor disponibilidad de glucosa al feto. 5. La HCG y el lactgeno placentario humano son la nicas hormonas sintetizadas por la placenta.

Cul de los siguientes antecedentes es considerado factor de riesgo del carcinoma de endometrio? 1. 2. 3. 4. 5. Multiparidad. Obesidad. Promiscuidad sexual. Endometritis. Abortos de repeticin.

180.

176.

Embarazada de 34 semanas que comienza con dolor uterino de aparicin brusca que se acompaa de ligero sangrado genital de color oscuro. A la exploracin el tero est doloroso y no se relaja entre contracciones. Los tonos cardacos fetales se escuchan con dicultad. Diagnstico: 1. Parto prematuro. 2. Placenta previa. 3. Desprendimiento de placenta normalmente inserta. 4. Coriamnionitis. 5. Aborto tardo inminente.

Ante una paciente de 42 aos que consulta por alopecia androide, hirsutismo de aparicin brusca, hipertroa de cltoris, amenorrea, sensacin de pesadez en hipogastrio y urgencia miccional, debemos descartar de forma inmediata: 1. 2. 3. 4. 5. Sndrome adrenogenital de aparicin tarda. Hirsutismo de origen perifrico. Tumor ovrico funcionante. Prolapso uterino en paciente perimenopusica. Pseudohermafroditismo masculino.

181.

177.

Acude a su centro una mujer primigesta de 35 aos por sensacin de dinmica regular. Se trata de una gestacin gemelar monocorial,

Mujer de 59 aos, menopasica desde hace 6, que acude a la consulta de ginecologa para su revisin anual. La exploracin fsica es rigurosamente normal, pero se decide realizarla una ecografa para seguimiento de unos miomas que han ido desapareciendo. En la ecografa ya no hay rastro de los miomas y el espesor del endometrio es de 9 mm. Qu hara a continuacin?

-23-

EXAMEN ENARM 2/10

1. Mandarla volver a revisin como vena haciendo. 2. Administracin de acetato de medroxiprogesterona en la segunda mitad del ciclo. 3. Histerectoma total con doble anexectoma. 4. Histeroscopia con biopsia dirigida. 5. Legrado diagnstico. 182. Paciente de 52 aos con ndulo mamario no doloroso de 1 cm en el cuadrante inferointerno de la mama derecha. La mamografa reere ndulo estrellado con microcalcicaciones agrupadas, compatible con carcinoma. La ecografa no es concluyente. Se realiza una puncin-aspiracin con aguja na (PAAF) que es informada como cambios propios de mastopata broqustica. Qu actitud debe tomar? 1. 2. 3. 4. 5. 183. Repetir la PAAF. Control mamogrco en 6 meses. Biopsia excisional. Tumorectoma y linfadenectoma. Mastectoma simple.

186.

Respecto a los anticonceptivos orales, seale cul de las siguientes armaciones es FALSA: 1. Los estrgenos disminuyen las LDL y aumentan las HDL. 2. Los progestgenos tienen un efecto aterognico. 3. Los estados de hipercoagulabilidad estn relacionados con la dosis de progestgeno. 4. Est incrementada la actividad de los factores II, VII, IX y X de la coagulacin. 5. Est aumentada la actividad brinoltica.

187.

Uno de los siguientes frmacos acta como antagonista del complejo glucoproteico IIb-IIIa: 1. 2. 3. 4. 5. Abciximab. Aspirina. Triusal Warfarina. Sulnpirazona.

188.

En los estudios iniciales de una amenorrea secundaria conviene comprobar la normalidad de los niveles sanguneos de: 1. 2. 3. 4. 5. TSH. ACTH. Dopamina. Estrona. Estriol.

Varn de 60 aos que acude repetidamente a su mdico de cabecera por un cuadro de dolores seos difusos que no responden a tratamiento con paracetamol 500 mg/8 horas. En la radiologa de columna se observan lesiones osteolticas en vrtebras y costillas. En el anlisis de sangre destaca entre otros datos una creatinina de 2,5 mg/dl. Considerando el diagnstico ms probable en este paciente, cul de los siguientes datos NO esperara encontrar usted? 1. 2. 3. 4. 5. VSG muy elevada. Descenso de beta-2-microglobulina. Descenso de inmunoglobulinas normales. Cadenas ligeras en orina. Acido rico: 9 mg/dl

184.

La lesin vulvar conocida como enfermedad de Bowen puede considerarse que es: 1. 2. 3. 4. 5. Una dermatosis escleroatrca. Una leucoplasia. Una lesin precancerosa. Un carcinoma in situ. Nada de lo anterior.

189.

Un paciente afecto de la enfermedad de Niemann-Pick presenta un cuadro de anemia grave que requiere transfusiones. Cul cree usted que es el mecanismo de la anemia que presenta este paciente? 1. 2. 3. 4. 5. Anemia Anemia Anemia Anemia Anemia hemoltica autoinmune. sideroblstica. perniciosa. mieloptsica. por dcit de hierro.

185.

Mujer de 55 aos, fumadora habitual, a la que se le descubre en una revisin anual un ndulo de 1,5 cm duro, jo e indoloro en el cuadrante superoexterno de la mama. En la exploracin fsica no se palpan adenopatas axilares ni supraclaviculares. La mamografa muestra imgenes sugestivas de malignidad. Se realiza biopsia intraoperatoria, cuyo resultado es carcinoma intraductal in situ. La actitud teraputica ms adecuada es: 1. Mastectoma simple. 2. Tumorectoma + radioterapia + biopsia del ganglio centinela. 3. Quimioterapia + radioterapia local + hormonoterapia si receptores estrognicos positivos. 4. Quimioterapia + hormonoterapia si receptores estrognicos positivos. 5. Mastectoma radical (Halsted).

190.

Una de las siguientes circunstancias NO alarga el tiempo de tromboplastina parcial activada: 1. 2. 3. 4. 5. Trombopenia severa. Hemolia B. Dcit de vitamina K. Tratamiento con heparina. Enfermedad de von Willebrand tipo III.

191.

Varn de 65 aos de edad sin antecedentes de inters, que consulta por cuadro de 6 meses de evolucin de astenia y prdida de 8 kilos de peso.

-24-

EXAMEN ENARM 2/10

En la exploracin presenta esplenomegalia dura a 10 cm y hepatomegalia a 5 cm del reborde costal. En la analtica se objetiva anemia con dacriocitos, 8.000 leucocitos/mm3, cuadro leucoeritroblstico, cifra de plaquetas normal, hiperuricemia y aumento de la LDH. El aspirado medular fue seco, por lo que realizas una biopsia de mdula sea en la que esperas encontrar: 1. Fibrosis reticulnica y colgena y pequeos focos de hipercelularidad hematopoytica. 2. Inltracin por leishmanias. 3. Aplasia medular. 4. Leucemia aguda mieloblstica promieloctica. 5. Enfermedad de Castleman. 192. Una de las siguientes relaciones entre leucemias agudas y marcadores es INCORRECTA: 1. 2. 3. 4. 5. 193. LAM M2-Peroxidasas. LAM M4-Esterasa inespecca. LAL L3-TdT. LAL L1-PAS. LAL comn-CALLA.

1. 2. 3. 4. 5. 196.

Esteroides. Sulfato de hierro. Acido folnico. Acido flico. Vitamina B12.

Paciente con enfermedad de Hodgkin que presenta adenopatas localizadas en las cadenas supraclavicular y occipital izquierda sin sntomas generales. Se encuentra en un estadio clnico: 1. 2. 3. 4. 5. I. IV. III. IIA. IIB.

197.

La incidencia del carcinoma de cavum es aproximadamente 100 veces ms frecuente en una de las siguientes reas geogrcas: 1. 2. 3. 4. Pases sajones al norte del trpico de Cncer. Regin china de Cantn. Costa africana de la cuenca mediterrnea. Cordillera andina, en alturas superiores a los 4.000 metros. 5. Franja costera del sur de Australia.

Un varn de 14 aos presenta un episodio de orinas oscuras y rpido desarrollo de sntomas de astenia, palidez e ictericia. Un anlis inmediato muestra Hb de 5 g/dl, reticulocitos, leucocitos y plaquetas normales, prueba de Coombs directa negativa y presencia de cuerpos de Heinz. Cual de los siguientes es el diagnstico ms probable: 1. 2. 3. 4. 5. Dcit de G-6-PD. Dcit de PK. Dcit de 5-nucleotidasa. Hemoglobinuria paroxstica nocturna. Anemia hemoltica por crioglobulinas.

198.

Un paciente acude a consulta por parlisis de la hemicara izquierda. A la exploracin fsica se observa desviacin de la comisura bucal hacia la derecha e imposibilidad para cerrar el ojo izquierdo. El test de Schirmer es positivo y la impedanciometra demuestra la presencia del reejo estapedial. Sin embargo el paciente reere disgeusia. La lesin se localiza a nivel de: 1. 2. 3. 4. 5. Ganglio geniculado. Protuberancia. Posterior a la segunda rodilla del facial. Posterior al oricio estilomastoideo. Conducto auditivo interno.

194.

Mujer de 69 aos asintomtica, que en un preoperatorio de cataratas presenta: leucocitos 30.000 mm3 con linfocitosis absoluta, Hb 12,6 g/dl, plaquetas 160.000 mm3 y en el frotis se objetivan abundantes sombras de Gumprecht. El inmunofenotipo es: CD5 +, CD 19, 20 y 23 +, FMC7 negativo. Su diagnstico es: 1. 2. 3. 4. 5. Leucemia prolinfoctica. Tricoleucemia. LNH de clulas del manto. Leucemia linftica crnica. Reaccin leucemoide.

199.

Varn de 58 aos, con antecedentes de cncer de cavum tratado mediante radioterapia, que presenta disfona (voz dbil) y disfagia alta, con crisis de tos y asxia en la deglucin. En la exploracin al sacar la lengua se va hacia el lado izquierdo y por laringoscopia se observa la cuerda vocal izquierda paralizada en posicin de abduccin. Cul es la causa de estos sntomas? 1. Invasin faringolarngea del tumor. 2. Recidiva local del tumor con extensin en base de crneo (agujero yugular). 3. Metstasis a distancia en pulmn. 4. Metstasis en ganglios mediastnicos, con compresin del nervio larngeo recurrente izquierdo. 5. Efectos secundarios de la radioterapia cervical, con lesin del recurrente izquierdo.

195.

Mujer de 52 aos de edad con artritis reumatoide que se encuentra actualmente controlada con tratamiento con metotrexato. En el hemograma se encuentra: Hb 10 g/dl, VCM 110, leucocitos 2.500 mm3, plaquetas 100.000 mm3, y en el frotis macroovalocitos e hipersegmentacin de neutrlos. El cuadro hematolgico de esta paciente precisar probablemente tratamiento con:

-25-

EXAMEN ENARM 2/10

200.

La reaccin de Weil-Flix, deteccin de anticuerpos contra los antgenos O de Proteus vulgaris, se utiliza para el diagnstico serolgico de determinadas enfermedades provocadas por Rickettsias. Sin embargo esta prueba es negativa en una de las siguientes enfermedades, selela: 1. 2. 3. 4. 5. Fiebre botonosa. Fiebre Q. Tifus endmico. Tifus epidmico. Fiebre manchada de las montaas rocosas.

cia, y en cuya placa de trax se objetiva un inltrado con broncograma areo en el segmento superior del lbulo inferior del pulmn derecho? 1. 2. 3. 4. 5. 204. Eritromicina. Amoxicilina-cido clavulnico. Penicilina G ms gentamicina. Doxiciclina. Cefotaxima ms aztreonam.

Dentro del grupo de los nemtodos (gusanos redondos) se incluyen todos EXCEPTO uno. Seale cul: 1. 2. 3. 4. 5. Ascaris lumbricoides. Trichuris trichiura. Strongyloides stercolaris. Echinococcus granulosus. Necator americanus.

201.

A.R.M. es un acionado a la espeleologa, que el pasado n de semana estuvo en las cuevas del Drac. Acude a urgencias por malestar general, tinte ictrico, lesiones purpricas en trax, abdomen y manos, junto con hemorragia conjuntival. En la exploracin encuentra hepatoesplenomegalia, y en la analtica aparece aumento de la creatinina y la urea. La radiografa trax demuestra derrame pleural bilateral sin cardiomegalia. Respecto a la patologa seale la FALSA: 1. El paciente padece el denominado sndrome de Weil. 2. El agente etiolgico es la Leptospira interrogans, espiroqueta anaerobia, mvil y con dos agelos pericitoplasmticos. 3. Para su diagnstico puede ser til aislar en lquidos o tejidos el agente en el medio de Ellinghausen-McCullough-Johnson-Harris. 4. El tratamiento de eleccin es la bencilpenicilina 1,5 millones U i.v. 4 veces al da. 5. No existe vector transmisor de esta enfermedad.

205.

Cul de los siguientes enterovirus ha sido implicado ms frecuentemente en casos de pericarditis y miocarditis infantiles? 1. Coxsackie B5. 2. Coxsackie A16. 3. Echo. 4. Enterovirus 70. 5. Polio. Una de las siguientes caractersticas NO es propia de los estreptococos del grupo Viridans: 1. Grupo heterogneo constituido por varias especies. 2. Forman parte de la ora oro-farngea. 3. Son la principal causa de endocarditis sobre vlvula nativa, en pacientes que no usan drogas por va parenteral. 4. Son resistentes a penicilinas. 5. Tienen especial habilidad para adherirse a supercies biolgicas.

206.

202.

Un paciente previamente sano es llevado a Urgencias con cefalea, ebre y signos menngeos. El estudio citobioqumico del LCR muestra pleocitosis monoctica, discreta elevacin de protenas, con glucosa normal. El examen del lquido con tincin de Gram y Ziehl, as como los cultivos bacterianos y para hongos, resultaron negativos. Qu pruebas microbiolgicas adicionales le practicara? 1. Deteccin de antgeno neumoccico en LCR. 2. Toma de muestras de faringe, heces y LCR para cultivo viral. 3. Cultivo en medios especiales para Haemophilus inuenzae. 4. Deteccin de antgeno criptoccico. 5. Deteccin de antgeno de Haemophilus inuenzae.

207.

Es correcto respecto a las infecciones posttransplante de mdula sea: 1. La vacunacin antineumoccica en estos enfermos est contraindicada. 2. La infeccin por CMV es caracterstica del primer mes. 3. La infeccin por VEB ocurre preferentemente por reactivacin del virus en linfocitos B del receptor. 4. El virus del herpes simple se aisla en los primeras semanas postransplante. 5. Los pacientes con neoplasias hematolgicas tienen menor riesgo de desarrollar neumona por Pneumocystis.

203.

Qu tratamiento considera ms adecuado para un anciano encamado y con atragantamientos frecuentes por un ictus previo, que acude al Hospital por un cuadro de ebre y somnolen-

208.

Qu tratamiento estara indicado en un drogadicto HIV+ con 500 CD4 que acude por ebre, presentando en la placa de trax inltracin

-26-

EXAMEN ENARM 2/10

multinodular en los segmentos apicales posteriores de los lbulos superiores y con Mantoux+? 1. Debe tratarse de la misma forma que los sujetos inmunocompetentes, aunque es conveniente evitar pautas de ciclo corto. 2. Debe tratarse de la misma forma que los sujetos inmunocompetentes con pautas de 6 o de 9 meses, ya que ambas han demostrado su ecacia. 3. En nuestro medio, debe hacerse un estudio previo de resistencias en caso de que la tuberculosis se d en inmunodeprimidos y en funcin de ste se elegir la pauta. 4. La duracin del tratamiento se modicar en funcin de la evolucin del paciente controlada mediante baciloscopias seriadas y en ningn caso ser inferior a 12 meses. 5. Es conveniente aadir a la pauta habitual un macrlido tipo claritromicina, ya que es muy ecaz contra el Mycobacterium avium intracellulare, que a menudo tambin coloniza los sujetos HIV+. 209. Una mujer de 34 aos acude al Servicio de Urgencia, porque desde hace 18 horas tiene cefalea, vmitos y fotofobia. En la exploracin la temperatura es de 38,5 C y tiene rigidez de nuca. La papila es normal y en la puncin lumbar el lquido cefalorraqudeo es de aspecto turbio. Cul es la conducta clnica ms apropiada en este momento? 1. Iniciar el tratamiento antibitico emprico inmediatamente. 2. Administrar tratamiento sintomtico nicamente, hasta no disponer de alguna informacin microbiolgica. 3. Realizar TC craneal. 4. Debe esperarse a los resultados del anlisis de clulas y protenas del LCR. 5. Poner 40 mg de prednisona i.v. para evitar secuelas cicatriciales. 210. Seale la FALSA respecto a la criptococosis: 1. Como prueba inmunolgica se utiliza la inmunouorescencia indirecta. 2. La meningitis es de curso subagudo. 3. Pueden formarse abscesos fros seos. 4. La criptococosis pulmonar, sobre todo en inmunocompetentes, puede curar sin tratamiento. 5. Es tpica la afectacin cutnea, que puede conformar masas heterogneas. 211. De las tcnicas serolgicas y microbiolgicas que se mencionan, cul elegira para la comprobacin diagnstica de la slis en perodo primario? 216. 1. TPI (Prueba de inmovilizacin de Nelson). 2. FTA (Prueba de inmunouorescencia indirecta). 214.

3. FTA-ABS (Variante de la anterior previa absorcin con la protena de Reiter). 4. VDRL. 5. Examen del exudado al campo oscuro. 212. Un paciente alcohlico, vagabundo, acude a Urgencias por sndrome febril de larga evolucin y deterioro del estado general. A la exploracin el paciente est sucio y descuidado y presenta un soplo sistlico en foco artico. Se realiza un ecocardiograma que demuestra la presencia de una verruga en la vlvula artica, a las dos semanas los hemocultivos continan negativos. Este paciente presenta una endocarditis por: 1. 2. 3. 4. 5. 213. Bartonella quintana. Streptococcus bovis. H. inuenzae. C. albicans. S. aureus.

Un enfermo desarrolla deterioro del estado general y lesiones cutneas bullosas y necrticas sobre una herida que el paciente presentaba previamente a un bao en el mar. El tratamiento de eleccin en dicho caso sera: 1. 2. 3. 4. 5. Desbridamiento quirrgico. Desbridamiento quirrgico ms cloxacilina. Tetraciclina. Eritromicina. Penicilina ms gammaglobulina antitetnica.

Usted atiende en Urgencias a un nio de 11 aos que presenta ebre de 38,5 C, malestar general, cefalea intensa, mialgias, dolor de garganta y vmitos. La auscultacin pulmonar muestra crepitantes aislados en lbulos inferiores. En la radiologa de trax se observa un patrn intersticial en ambos lbulos inferiores y atelectasias subsegmentarias. Cul es su sospecha diagnstica? 1. 2. 3. 4. 5. Meningitis meningoccica. Neumona por neumococo. Infeccin por Mycoplasma pneumoniae. Neumona por Pneumocystis carinii. Leptospirosis.

215.

La causa ms frecuente de ebre persistente o recurrente durante el tratamiento de una endocarditis infecciosa es: 1. 2. 3. 4. 5. Rgimen antibitico equivocado. Aparicin de resistencia durante el tratamiento. Embolias spticas. Absceso miocrdico. Fiebre medicamentosa.

Acude a urgencias un paciente por presentar crisis de angina ms frecuentes y con menos esfuerzo de lo que le era habitual, desde hace

-27-

EXAMEN ENARM 2/10

un mes. Estaba en tratamiento con nitroglicerina sublingual a demanda y betabloqueantes. Se decide ingresarle para ponerle tratamiento con antianginosos, AAS y heparina. Tras 48 horas del ingreso se encuentra asintomtico, cul sera la actitud diagnstico-teraputica ms correcta? 1. Aumentar las dosis de los frmacos antianginosos que tena como tratamiento de base. 2. Realizar coronariografa y revascularizacin segn los hallazgos. 3. Realizar ergometra antes del alta y, si sta muestra datos de mal pronstico, hacer coronariografa. 4. Asociar al tratamiento antianginoso tratamiento anticoagulante. 5. Dar de alta y remitir a la semana siguiente al cardilogo para hacer coronariografa. 217. El cuarto ruido cardaco aparece coincidiendo con: 1. 2. 3. 4. 5. 218. Contraccin ventricular. Contraccin auricular. Apertura de la vlvula mitral. Apertura de la vlvula artica. Llenado ventricular rpido.

4. Estudio electrosiolgico. 5. ECG tipo Holter. 220. Pueden considerarse combinaciones adecuadas de antihipertensivos las siguientes EXCEPTO: 1. Calcioantagonistas con betabloqueantes. 2. Calcioantagonista con inhibidor de la convertasa de la angiotensina. 3. Betabloqueante con diurtico. 4. Calcioantagonista con diurtico. 5. Inhibidor de la convertasa de la angiotensina con diurtico. 221. Paciente de 62 aos, fumador e hipertenso, que ha ingresado en el servicio de cardiologa por presentar clnica compatible con un IAM, que se confirma como infarto anterior bastante extenso. Ha sido tratado con mrficos, AAS y tromblisis con estreptoquinasa. Ha evolucionado favorablemente sin tener ninguna complicacin. El sptimo da de evolucin se le hace una ergometra que resulta negativa. Cul de las siguientes medidas de prevencin secundaria NO es adecuada? 1. Tratamiento con anticoagulantes orales, ya que ha tenido un infarto extenso, que se debe mantener de por vida. 2. Control de la TA, colesterol y abandono del hbito tabquico. 3. Tratamiento de su HTA con betabloqueantes preferentemente. 4. Tratamiento con estatinas para mantener un colesterol LDL < 100. 5. Administracin de captopril para intentar reducir el remodelado ventricular. 222. Paciente que consulta por dolor intenso en el centro del trax, que se extiende a la espalda, agudo y que se incrementa con la tos, pero se alivia al inclinarse hacia delante. En la auscultacin se escucha un roce distante de espiracin con el paciente sentado. El ECG demuestra elevacin de los segmentos ST de V2 a V6. Este cuadro NO est producido por: 1. 2. 3. 4. 5. 223. Virus Coxackie B. Lupus eritematoso sistmico. Mononucleosis infecciosa. Tetraciclinas. Procainamida.

Varn de 46 aos que presenta disnea de moderados esfuerzos desde hace cuatro meses. Como sintomatologa acompaante reere sensacin de palpitaciones espordicas desde hace ms de un ao y dolor torcico inespecco que no se relaciona con nada. En la exploracin encontramos frecuencia cardaca de 84 lpm, TA de 150/75 mmHg, pulso hiperdinmico, latido de la punta hiperdinmico en el sexto espacio intercostal izquierdo desplazado lateralmente. Auscultacin: ritmo sinusal, soplo diastlico en tercer espacio intercostal izquierdo que aumenta con la realizacin de ejercicio fsico isomtrico. Cul es su diagnstico de sospecha? 1. 2. 3. 4. 5. Insuciencia pulmonar. Estenosis mitral. Insuciencia mitral. Estenosis artica. Insuciencia artica.

219.

Paciente de 22 aos que acude a consulta por presentar episodios espordicos de palpitaciones a los que se le asocia ansiedad. Se le practica un ECG en el que se observan ondas P normales, con intervalo PR de 0,11 seg, QRS de 0,14 seg con un empastamiento inicial. Qu tcnica complementaria utilizara para completar el diagnstico? 1. Tilt test. 2. Ecocardiograma transesofgico. 3. Prueba de esfuerzo.

Paciente de 70 aos que ingresa en la unidad coronaria por un episodio de dolor torcico diagnosticado de IAM anterior. Entre sus antecedentes destaca la existencia de diabetes mellitus y que es ex-fumador de 25 cigarrillos/da desde hace 15 aos. A las pocas horas comienza con disnea de moderada intensidad e hipotensin, acompaada de mala perfusin perifrica.

-28-

EXAMEN ENARM 2/10

Cul de las siguientes medidas teraputicas NO estara indicada en este caso? 1. Betabloqueantes para disminuir la isquemia y la tasa de rotura cardiaca 2. Diurticos y vasodilatadores como frmacos de eleccin. 3. Dopamina o dobutamina asociadas a lo anterior si el paciente no mejora. 4. Realizacin de ecocardiograma para descartar complicaciones mecnicas 5. Si no responde al tratamiento mdico se puede recurrir al baln de contrapulsacin intraartico. 224. Seale cul de los siguientes frmacos o maniobras teraputicas tiene MENOS importancia en el tratamiento de un paciente en edema agudo de pulmn, con taquicardia sinusal a 120 lpm y tensin arterial de 120/80 mmHg: 1. 2. 3. 4. 5. 225. Digoxina. Oxgeno. Morna. Vasodilatadores. Furosemida.

227.

Paciente de 65 aos visto por el cardilogo con anterioridad por el seguimiento de una angina estable de moderados esfuerzos, que se controlaba adecuadamente hasta hace dos meses con propanolol y nitroglicerina sublingual durante las crisis. Actualmente viene a urgencias por nuevo cuadro anginoso, reere que las crisis se estn haciendo ltimamente ms frecuentes y que aparecen con menos esfuerzo. En la exploracin fsica se observa TA de 140/85 mmHg, 90 lpm, con el resto dentro de lo normal. En el ECG basal tiene una onda T negativa en V5-V6 y descenso del ST en el periodo sintomtico. Qu tratamiento le propondra? 1. Ingreso hospitalario y coronariografa urgente tras control del dolor con nitroglicerina sublingual. 2. Ingreso hospitalario, reposo y tratamiento con AAS, clopidogrel y heparina de bajo peso molecular, tratamiento antianginoso betabloqueantes y nitroglicerina y realizacin de prueba de esfuerzo pronstica 48 horas despus de controlado el dolor. 3. Ingreso en la unidad coronaria instaurando trombolticos. 4. Control del dolor con nitroglicerina sublingual y betabloqueantes, y tras ello remitir al cardilogo para reajuste del tratamiento. 5. Ingreso hospitalario, tratamiento con AAS, heparina no fraccionada y control del dolor y tras ello, alta hospitalaria con tratamiento y realizacin de prueba de esfuerzo para estadicacin del riesgo de forma ambulatoria.

Paciente de 59 aos que est ingresado por presentar un IAM de localizacin inferior hace seis das que ha cursado sin complicaciones. Actualmente se encuentra asintomtico y la exploracin fsica es normal, salvo por una TA de 160/95 mmHg. Para el manejo adecuado de este paciente tras el IAM, se debe hacer todo lo siguiente EXCEPTO: 1. Valoracin del miocardio en riesgo a los 7-10 das del infarto mediante ergometra. 2. Tratamiento con AAS, 125-300 mg/da para prevenir nuevos episodios de isquemia. 3. Administracin de estatinas para mantener el colesterol total en cifras menores de 200 mg/dl. 4. Administracin de betabloqueantes ya que mejoran el pronstico, sobre todo porque el paciente presenta HTA. 5. Administracin de antagonistas del calcio como prolaxis de arritmias durante las 2-3 semanas que dura el proceso de cicatrizacin. 228.

Cul de las siguientes armaciones dira usted que es cierta en relacin a las necesidades energticas en la enfermedad? 1. A medida que aumenta el consumo de oxgeno, tambin lo hace la termognesis inducida por la dieta. 2. Aun en las enfermedades graves, las necesidades energticas de la mayora de los pacientes son slo algo superiores a las de las personas sanas. 3. La energa necesaria para la actividad fsica no debe tenerse en cuenta para al calcular las necesidades calricas de los pacientes enfermos. 4. El clculo del ndice metablico en reposo de los pacientes enfermos, debe hacerse de manera peridica, individual y directa, a partir de la determinacin de su consumo de oxgeno. 5. La malabsorcin es una causa habitual de aumento de las necesidades metablicas en casi todas las enfermedades.

226.

Una mujer de 82 aos ha presentado en cuatro ocasiones, en la ltima semana, episodios de prdida de conciencia. Un ECG muestra ritmo sinusal a 50 lpm, y una pausa sistlica de 2,5 segundos. El siguiente paso a realizar ser: 1. Efectuar prueba de esfuerzo. 2. La monitorizacin ambulatoria del ritmo cardaco (Holter) durante 24 horas. 3. Iniciar directamente tratamiento con isoprenalina, sin ms. 4. Implantar un marcapasos ventricular permanente a demanda. 5. Colocar un marcapasos temporal.

229.

Mujer de 49 aos con osteoporosis consulta por la aparicin en el ltimo ao de 2 episodios de litiasis renal de oxalato clcico. Los antecedentes familiares de litiasis son negativos. El

-29-

EXAMEN ENARM 2/10

estudio de su osteoporosis revela un aumento de la eliminacin urinaria de hidroxiprolina e hidroxilisina. La analtica sangunea muestra un aumento de fosfatasa alcalina, Ca = 12,3 mg/dl y P = 2,5 mg/dl. En la Rx de manos se observa una imagen de resorcin subperistica a nivel de algunas falanges. Qu enfermedad debera descartar en primer lugar? 1. 2. 3. 4. 5. 230. Enf. de Paget. Osteocondromas mltiples en las falanges. Hiperparatiroidismo. Hipercalcemia tumoral. Hipercalciuria familiar.

3. Hipercalcemia, hipopotasemia e hipofosfatemia. 4. Presencia de metstasis al diagnstico. 5. Aumento de la concentracin plasmtica de VIP y volumen de heces de al menos 1l/ da. 233. Varn de 35 aos, enolismo moderado, consulta por hiperpigmentacin ocular, as como hipertricosis malar, ampollas y erosiones en dorso de manos a mnimos traumatismos, sensaciones punzantes de la piel con la exposicin solar de reciente aparicin. Qu orientacin diagnstica le sugiere? 1. 2. 3. 4. 5. 234. Porria aguda intermitente. Porria cutnea tarda. Dermatomiositis. Cloasma. Toxicodermia.

Un varn de 60 aos consulta por un sndrome cisttico rebelde y Vd. comprueba por tacto rectal que su prstata es normal. Qu diagnstico debe sospechar con prioridad? 1. Un carcinoma in situ vesical. 2. Un carcinoma prosttico latente de bajo estadio (A1-A2...). 3. Una prostatitis por Chlamydia. 4. Una estenosis de uretra. 5. Una hiperplasia del lbulo medio prosttico subcervical.

231.

Varn de 6 aos es estudiado por presentar infecciones recidivantes mucocutneas por cndida. En la exploracin se aprecia una hiperpigmentacin en plantas y palmas, as como otras reas de vitligo parcheadas. En la analtica destacan: Na+ 128, K+ 5,9. Creatinina normal. Protenas totales: 5 g/dl (normal). Calcio total: 7,5 mg/dl. Fsforo total: 6,5 mg/dl. Glucemia: 60 mg/dl. TA: 80/50. Cul sera el diagnstico correcto? 1. Episodio de desnutricin con hipocalcemia e hipo TA secundarias en pacientes con infecciones fngicas crnicas. 2. Sndrome de SCHMIDT. 3. Sndrome poliglandular autoinmune tipo I. 4. Sndrome de Werner. 5. Pseudohipoparatiroidismo.

Seale la cierta en relacin a las vitaminas: 1. Las fuentes de vitamina A son los alimentos de origen animal, fundamentalmente el hgado, el huevo, la leche y sus derivados, estando prcticamente ausente en los vegetales. 2. La biotina se obtiene a partir de la levadura, productos animales y por sntesis intestinal, y se inactiva por la avidina de la clara de huevo. 3. El exceso de rivoavina o vitamina B2 produce rubor y prurito cutneo por su efecto vasodilatador. 4. Tanto el exceso como la carencia de piridoxina o vitamina B6 pueden ser causa de neuropata. 5. La vitamina E es un importante antioxidante y estabilizador de membranas, y sus fuentes son fundamentalmente alimentos de origen vegetal. Varn de 31 aos acude a consulta por mltiples ppulas amarillentas en la cara. La biopsia revela una hiperplasia sebcea y multiples adenomas sebceos. El paciente est siendo visto en el Servicio de Digestivo por el hallazgo de sangre oculta en heces realizado como screening de carcinoma colon debido a que su padre y su abuelo murieron de ello. Qu sndrome cree que presenta este paciente? 1. 2. 3. 4. 5. Sind. non poliposis. Esclerosis tuberosa. Sind. de Turcot. Sind. de Torre. Sind. de Gardner.

235.

232.

Paciente que presenta desde hace algunos meses molestias vagas abdominales, sensacin de plenitud gstrica precoz, diarrea acuosa y debilidad muscular marcada. En un ECG que se le realiz recientemente por los equipos mdicos de su empresa como estudio rutinario destaca la presencia de onda U y aplanamiento de la onda T. Cul de las siguientes cree que sera MENOS probable en este caso? 1. Masa pancretica detectada en la ecografa abdominal. 2. Hiperglucemia debida a la destruccin insular pancretica producida por la proliferacin de la masa tumoral.

236.

La hipertensin intracraneal benigna se ha asociado a intoxicacin con los siguientes frmacos, EXCEPTO uno. Selelo: 1. 2. 3. 4. 5. Vitamina A. cido acetilsaliclico. Tetraciclinas. Nitrofurantona. Indometacina.

-30-

EXAMEN ENARM 2/10

237.

Tras la administracin de tiamina, el paciente con encefalopata de Wernicke comienza la recuperacin clnica siguiendo una secuencia en la mejora de los sntomas. Seale cul de los siguientes enunciados reeja la secuencia habitual de recuperacin: 1. 2. 3. 4. 5. 1 1 1 1 1 oftalmoparesia, 2 cuadro confusional, 3 ataxia. cuadro confusional, 2 ataxia, 3 oftalmoparesia. oftalmoparesia, 2 ataxia, 3 cuadro confusional. ataxia, 2 oftalmoparesia, 3 cuadro confusional. cuadro confusional, 2 oftalmoparesia, 3 ataxia.

4. Dolor de difcil localizacin, urente. 5. La conservacin de la percepcin posicional articular, con prdida de otras funciones. 242. Un paciente con antecedente de traumatismo cervical leve presenta un episodio de amaurosis fugax por el ojo izquierdo. Posteriormente desarrolla ptosis y miosis izquierda, dolor laterocervical y dcit motor hemicorporal derecho. Cul de los siguientes es el diagnstico ms probable? 1. 2. 3. 4. 5. 243. Arteritis de la temporal. Robo de la subclavia. Diseccin de la cartida interna. Trombosis venosa. Enfermedad de Moya-Moya.

238.

Una debilidad para la dorsiexin y eversin del pie y extensin del dedo grueso, conservando la fuerza para la inversin de dicho pie, asociada con una disminucin de la sensibilidad en la cara lateral de la pierna y dorso del pie, con qu estructura nerviosa la relacionara? 1. 2. 3. 4. 5. Nervio peroneal. Nervio tibial posterior. Nervio femoral lateral cutneo. Nervio crural. Raz S1.

Un adolescente comienza a presentar de forma progresiva coreoatetosis, distona, ataxia y deterioro intelectual. El examen corneal con lmpara de hendidura es negativo y no hay antecedentes de patologa neonatal. El diagnstico ms probable es: 1. 2. 3. 4. 5. Enfermedad Enfermedad Enfermedad Enfermedad Kernicterus. de de de de Hallevorden-Spatz. Wilson. Pick. Lafora.

239.

Mujer de 28 aos de edad que reere visin doble y cada de los prpados de unos meses de evolucin. A estos sntomas iniciales se les aadi dicultad para tragar y debilidad en las extremidades inferiores. La paciente dice encontrarse unos das mejor que otros y mucho mejor al levantarse por la maana que por la noche. En la exploracin se puso de maniesto fatigabilidad muscular en los msculos de la cara, extraoculares y en los de las extremidades; no presentaba alteraciones sensitivas ni amiotroas y los reejos musculares profundos estaban conservados. Cul es el diagnstico ms probable? 1. 2. 3. 4. 5. Botulismo. Sndrome miastnico de Lambert-Eaton. Miastenia gravis. Esclerosis mltiple. Miopata hipertiroidea.

244.

Al explorar a un paciente afecto de una lesin de la neurona motora cortical NUNCA encontraremos: 1. 2. 3. 4. 5. Hipertona muscular. Fasciculaciones. Signo de Babinski. Hiperreexia. Hemiparesia, cuadriparesia o paraparesia.

245.

240.

En la edad infantil los tumores cerebrales ms frecuentes son: 1. 2. 3. 4. 5. Gliomas. Craneofaringiomas. Meduloblastomas. Pinealomas. Metstasis. 246.

Un paciente de 15 aos presenta desde hace 1 ao mioclonas matutinas en exin, acompaadas en los ltimos 6 meses de 2 crisis tonico-clnicas generalizadas inducidas por la deprivacin de sueo. El EEG demuestra complejos punta-onda generalizados, con actividad de fondo bien estructurada. Seale el tratamiento de primera eleccin: 1. 2. 3. 4. 5. Gabapentina. Vigabatrina. Carbamacepina. cido valproico. Primidona.

241.

Los datos siguientes indican lesin intramedular, EXCEPTO: 1. El dcit sensitivo sacro. 2. Los signos corticoespinales. 3. Mnimas alteraciones del LCR.

Los hematomas agudos postraumticos extraaxiales, epidural y subdural, tienen similitudes y diferencias importantes. Seale cul de las siguientes caractersticas es claramente ms propia del hematoma subdural que del epidural:

-31-

EXAMEN ENARM 2/10

1. 2. 3. 4. 5. 247.

Fractura asociada. Intervalo lcido. Forma de semiluna en la TC. Sangrado de arteria menngea media. Individuos jvenes en accidente de trco.

sentirse hinchado y con pesadez despus de las comidas. No ha perdido peso y no reere ningn otro dato de inters. Respecto al tratamiento de la enfermedad que sospecha indique la FALSA: 1. Si el paciente reconoce algn alimento que exacerbe los sntomas, es aconsejable que lo retire de su dieta. 2. Es fundamental explicar claramente al paciente en qu consiste su enfermedad y tranquilizarle respecto al pronstico. 3. El uso de enzimas pancreticas puede estar indicado para evitar la maldigestin. 4. Algunos pacientes se benecian del uso de anticolinrgicos, sobre todo los que presentan dolor abdominal. 5. Antidiarreicos como loperamida pueden estar indicados de forma temporal. 252. Un enfermo con cirrosis heptica, con ascitis importante, que est ingresado desde ayer en el servicio de Digestivo, se encuentra en encefalopata heptica grado 3. Cul de las siguientes actitudes le parece INCORRECTA? 1. 2. 3. 4. 5. Realizar al enfermo un tacto rectal. Realizar una paracentesis diagnstica. Administrar paramomicina. Incrementar la dosis de furosemida. Iniciar un tratamiento con enemas cada 12 horas.

Mujer de 18 aos con 6 episodios al mes de cefalea hemicraneal, de un da de duracin, pulstil, acompaada de vmitos, fotofobia y sonofobia y con examen fsico normal. El tratamiento preventivo de eleccin, entre los siguientes, es: 1. 2. 3. 4. 5. Carbamacepina. Carbonato de litio. Ergotamina. Propranolol. Sumatriptn.

248.

Respecto a la denominada encefalopata de Binswanger, una de las siguientes armaciones NO es correcta. Indquela: 1. 2. 3. 4. 5. Es una forma de demencia no vascular. Cursa con desmielinizacin de la sustancia blanca. Se asocia a infartos lacunares. Suele ocurrir en pacientes con hipertensin arterial. La localizacin de la lesiones es periventricular.

249.

Paciente varn de 60 aos de edad que presenta dolor abdominal difuso que ocasionalmente se extiende desde el epigastrio a la espalda, que mejora algo al exionar el tronco hacia delante. El dolor se acompaa de la presencia de ictericia, prurito, orinas colricas y prdida de peso. A la exploracin se palpa una vescula biliar aumentada de tamao y signos de tromboebitis en las piernas. Analticamente est elevada la fosfatasa alcalina, la gamma GT y la bilirrubina. Cul ser el origen del cuadro de este paciente? 1. 2. 3. 4. 5. Tumor de Klastkin. Cncer de pncreas. Pancreatitis crnica. Hepatocarcinoma. Colecistitis aguda.

253.

Si en un paciente cirrtico encuentra una fosfatasa alcalina srica desproporcionadamente elevada en relacin con otras pruebas de funcin heptica, es un indicio de: 1. 2. 3. 4. 5. Enfermedad de Wilson. Cirrosis biliar primaria. Hemocromatosis. Cirrosis alcohlica. Carcinoma heptico inltrante.

254.

Cul de las siguientes pruebas diagnsticas de malabsorcin suele ser normal en los sujetos que padecen un sobrecrecimiento bacteriano? 1. Cuanticacin de la grasa fecal (24 h). 2. Fase II de la prueba de Schilling (administracin de factor intrnseco ms vitamina B12. 3. Prueba de absorcin de la D-xilosa. 4. Prueba del hidrgeno espirado al administrar lactulosa. 5. Cultivo del aspirado yeyunal con recuento de colonias.

250.

Varn de 55 aos fumador y bebedor importante, sin otros antecedentes, que acude a consulta por disfagia para slidos progresiva de pocos meses de evolucin junto con prdida de 10 kg de peso y astenia. Su sospecha diagnstica es: 1. 2. 3. 4. 5. Acalasia. Esclerodermia. Carcinoma esofgico. Anillo esofgico inferior. Estenosis pptica.

255.

La poliposis adenomatosa familiar y el sndrome de Gardner se han puesto en relacin con la alteracin de un gen de los siguientes, selelo: 1. C-yes. 2. C-src.

251.

Varn de 32 aos que acude por alteracin del trnsito intestinal junto con episodios paroxsticos de dolor abdominal de tipo clico. Reere

-32-

EXAMEN ENARM 2/10

3. APC. 4. C-fos. 5. C-mos. 256. En un paciente de 70 aos se ha detectado por colonoscopia una tumoracin en colon descendente, estenosante y con resultado histolgico de adenocarcinoma moderadamente diferenciado. Las pruebas complementarias no demuestan diseminacin del proceso neoplsico. Cul debera ser el tratamiento a plantear? 261. 1. Quimioterapia con 5-uoracilo y levamisol. 2. Reseccin quirrgica y tratamiento quimioterpico segn resultado del estudio. 3. Radioterapia combinada con quimioterapia y posterior reseccin quirrgica. 4. Reseccin quirrgica y tratamiento radioterpico complementario. 5. Practicar reseccin quirrgica slo en caso de existir signos clnicos de oclusin intestinal. 257. La enteroparasitosis que origina cuadros de mal-absorcin y afecta con ms frecuencia en inmunodeciencias, especialmente en los dcit aislados de IgA, debe tratarse con: 1. 2. 3. 4. 5. 258. Praziquantel. Bithionol. Metronidazol. Niclosamina. Espiramicina.

2. Es la causa de hemorragia digestiva baja en el 25% de los pacientes mayores de 50 aos, siendo ms frecuente en pacientes con estenosis artica. 3. La arteriografa selectiva de la arteria mesentrica inferior es diagnstica. 4. El tratamiento inicial es hemicolectoma derecha. 5. Con frecuencia es un hallazgo casual durante una laparotoma. Nia de 3 aos de edad que acude a su pediatra por estancamiento ponderal desde los 2 aos y 3 meses de vida. Realiza 6-8 deposiciones al da, abundantes, pastosas y brillantes. Los padres reeren que es mala comedora y que siempre est enfadada y llorando por todo. A la exploracin llama la atencin una importante distensin abdominal, as como un aspecto distrco de las extremidades. En relacin a la enfermedad que usted sospecha seale la armacin CORRECTA: 1. El diagnstico es de enfermedad celaca y no es necesaria la realizacin de ms pruebas complementarias. 2. La actitud inicial ms adecuada sera la determinacin de los anticuerpos antitransglutaminasa. 3. La gravedad del cuadro clnico expuesto hace necesaria la realizacin de una biopsia intestinal urgente. 4. La biopsia intestinal se debe realizar a nivel del leon. 5. La elevacin de transaminasas descarta enfermedad celaca. 262. Varn de 46 aos con antecedentes de haber padecido condilomas acuminados en regin perianal, que acude al mdico por presentar dolor perianal con la defecacin. A la exploracin presenta una masa indurada ulcerada de 3 cm de dimetro en el conducto anal, que se biopsia y el resultado es de un carcinoma de clulas escamosas del conducto anal. Ante esta situacin el tratamieto actualmente ms adecuado sera: 1. Escisin local de la tumoracin sin terapia adyuvante. 2. Escisin local de la tumoracin asociada a radioterapia y quimioterapia. 3. Amputacin abdominoperineal sin terapia adyuvante. 4. Amputacin abdominoperineal asociada a radioterapia y quimioterapia. 5. Radioterapia y quimioterapia nicamente. 263. Un anciano de 87 aos es trado al servicio de Urgencias de su hospital por presentar un cuadro compatible con una obstruccin intestinal. Seale cul de las siguientes es la entidad con

Varn de 63 aos que acude al servicio de Urgencias con un cuadro de rectorragia masiva, con discreto dolor abdominal difuso. Cul ser la causa ms frecuente ante este cuadro? 1. 2. 3. 4. 5. Cncer de colon izquierdo. Colitis ulcerosa. Cncer de colon derecho. Angiodisplasia de colon. Enfermedad diverticular del colon.

259.

Un paciente con infeccin por VIH y 100 linfocitos T CD4/mm3 consulta por disfagia y ebre de tres das de evolucin. En la exploracin nicamente destaca la presencia de leucoplasia oral peluda y muguet oral. Cul de estos tratamientos empricos sera ms adecuado para esta paciente? 1. 2. 3. 4. 5. Aciclovir. Ganciclovir. Metronidazol. Fluconazol. Anfotericina B.

260.

La angiodisplasia de colon: 1. Es frecuente en pacientes con fracaso renal crnico.

-33-

EXAMEN ENARM 2/10

mayores probabilidades de ser la responsable de esta situacin: 1. 2. 3. 4. 5. 264. Apendicitis aguda. Cncer de colon. leo biliar. Invaginacin intestinal. Vlvulo de sigma.

267.

Qu actitud tomara ante un paciente ingresado por presentar un infarto de miocardio y que presenta una taquicardia ventricular sostenida, aunque sin deterioro hemodinmico? 1. 2. 3. 4. 5. Cardioversin elctrica. Lidocana (1 o 2 dosis de 50 a 100 mg). Masaje cardaco. Bretilio. Verapamil i.v.

Con respecto a la aparicin de ebre (temperatura superior a 38 C) en el postoperatorio, y suponiendo que no haya ninguna causa preoperatoria que justique la hipertermia, seale cul de las siguientes armaciones NO es cierta: 1. La aparicin de ebre en el postoperatorio es muy frecuente. 2. La causa ms frecuente de ebre en las primeras 24 horas del postoperatorio es la atelectasia. 3. La ebre por infeccin de la herida quirrgica suele aparecer entre el 7 y el 10 das del postoperatorio. 4. Las reacciones febriles entre las 24 y 72 horas postoperatorias generalmente se deben a infecciones urinarias. 5. Un absceso intraabdominal o una fuga anastomtica suelen causar ebre entre el 4 y el 7 das del postoperatorio.

268.

El registro de los potenciales elctricos generados por el corazn se denomina electrocardiografa. Seale la FALSA en relacin a l: 1. Tanto la onda P como los componentes del complejo QRS son ondas de despolarizacin. 2. La onda T es una onda de repolarizacin. 3. Durante la despolarizacin se pierde el potencial negativo normal dentro de las bras. 4. La onda T suele ser de procedencia auricular ya que la onda T ventricular suele quedar enmascarada por el complejo QRS. 5. Cuando el msculo ventricular est completamente polarizado o completamente despolarizado no se puede registrar ningn potencial proveniente de l.

269.

265.

Varn de 45 aos diabtico con retinopata e insuciencia renal asociada. Consulta por presentar ebre elevada y diarrea de 3 das de evolucin, de 10 a 12 deposiciones al da con moco y tenesmo. A la exploracin fsica se encuentra deshidratado con TA 90/70 a 100 lpm y con hiperperistaltismo intestinal. En la analtica destaca glucosa 420 mg/dl, 21.500 leucocitos, creatinina 2,3 mg/dl y test positivo de leucocitos fecales. Las siguientes medidas seran recomendables, EXCEPTO: 1. 2. 3. 4. 5. Hemocultivos. Tratamiento con quinolonas. Coprocultivo. Fluidoterapia i.v. Trnsito baritado intestinal.

Paciente de 64 aos que acude a consulta reriendo una historia de dicultad respiratoria, progresiva con el ejercicio, a la que en los ltimos meses se ha unido dolor opresivo centrotorcico irradiado al cuello que aparece a los cinco minutos de un ejercicio y cede con el reposo. En la ltima semana ha perdido dos veces el conocimiento mientras corra a coger el autobs. Respecto a la patologa que probablemente padece nuestro paciente seala la FALSA: 1. El pulso arterial probablemente muestre las caractersticas de pulso Corrigan. 2. En la auscultacin probablemente destaque un soplo sistlico irradiado a cartidas. 3. Puede palparse un frmito sistlico en la base del corazn. 4. Puede auscultarse un cuarto tono en el pex. 5. La hipertensin arterial signicativa es poco probable si la patologa que padece es muy marcada.

266.

Paciente diagnosticado de estenosis mitral, que acude a Urgencias por gran disnea de reposo, despus de haber incumplido su tratamiento habitual. La presin arterial es de 150/90 mmHg y est en brilacin auricular a 150 lat/ min. Qu medidas teraputicas recomendara? 1. Oxgeno, morna, furosemida y dopamina. 2. Oxgeno, morna, furosemida, digoxina y reposo. 3. Vasodilatadores, digital y dopamina. 4. Reposo absoluto, furosemida y nitroprusiato. 5. Reposo absoluto, furosemida y vasodilatadores.

270.

Un mdico de un conocido hospital madrileo nota al poco de levantarse, un dolor de instauracin brusca que ms tarde describir como muy intenso y desgarrador, localizado en la parte central del trax y el epigastrio, que se irradiaba al cuello y a la zona lumbar. Se tumba para descansar y el reposo no calma sus molestias. Recuerda, mientras su familia avisa a la ambulancia, que ltimamente sus cifras tensionales eran de 190/110 mm Hg. Cul cree que ser su autodiagnstico?

-34-

EXAMEN ENARM 2/10

1. 2. 3. 4. 5. 271.

Infarto de miocardio. Tromboembolismo pulmonar. Aneurisma disecante de la aorta. Aneurisma artico en expansin. Pericarditis.

274.

Un varn de 38 aos de edad presenta un episodio de dolor torcico retroesternal y precordial izquierdo que se extiende a la espalda y al borde del trapecio y que aumenta a la inspiracin. En el ECG se objetiva ascenso difuso del segmento ST de concavidad superior con descenso en aVR. En la analtica se demuestra elevacin de la fraccin MB de la creatinfosfoquinasa. Cul de las siguientes recomendaciones NO le parece adecuada? 1. 2. 3. 4. 5. Aspirina a dosis altas. Reposo en cama. Anticoagulantes orales. Vigilancia de las presiones arterial y venosa. Realizar un ecocardiograma. 275.

Un paciente de 59 aos de edad ha sido diagnosticado recientemente de hipertensin arterial. El paciente se encuentra asintomtico, la exploracin fsica no demuestra hallazgos de importancia, y tanto la radiografa de trax como el electrocardiograma son tambin normales. Seale lo cierto con respecto a este paciente: 1. Tiene menos posibilidades que un paciente de 45 aos de tener hipertensin arterial secundaria. 2. Si tuviera hipertensin arterial vasculorrenal, probablemente se debera a displasia bromuscular de las arterias renales. 3. Habra que empezar desde el principio con tratamiento farmacolgico. 4. Tiene ms probabilidades de tener lesiones coronarias que un sujeto no hipertenso. 5. Los diurticos son actualmente el tratamiento de eleccin de la hipertensin arterial. El principio de Fick se usa para: 1. 2. 3. 4. Determinar presiones. Determinar el gasto cardaco. Determinar gradientes. Determinar la gravedad de una valvulopata regurgitante. 5. Determinar el QP/QS.

272.

Una paciente de 48 aos presenta palpitaciones desde hace unas horas, motivo por el que acude a Urgencias. En el electrocardiograma llama la atencin un ritmo irregular a 140 lpm, con ausencia de ondas P y lnea de base sustituida por un temblor, el QRS es de 0,11 segundos. No se encuentran sntomas ni signos de inestabilidad hemodinmica. En este paciente es FALSO: 1. Si el paciente empieza a desestabilizarse estara indicado realizar cardioversin elctrica. 2. Antes de cardiovertir farmacolgicamente es necesario dar frmacos que disminuyen la conduccin del nodo AV para evitar una respuesta ventricular ms rpida y peor tolerada. 3. Si la FA es de ms de seis meses de duracin es difcil conseguir la cardioversin. 4. Tambin es difcil la cardioversin con aurculas de tamao mayor de 5 cm de dimetro. 5. Si la FA es de larga duracin es necesario antiagregar al paciente desde 2-3 semanas antes de la cardioversin para evitar posibles tromboembolismos.

276.

Respecto a la trombosis venosa profunda, la actitud ms importante es intentar evitarla en situaciones de alto riesgo, para lo cual slo uno de los mtodos siguientes est CONTRAINDICADO: 1. 2. 3. 4. 5. Media elstica. Heparina clcica subcutnea a dosis bajas. Reposo en cama. Anticoagulacin oral. Elevacin de miembros inferiores.

277.

Respecto a las complicaciones del trasplante de pulmn es FALSO: 1. El 10-15% sufren estenosis, que pueden resolverse por brobroncoscopia. 2. La primera causa de mortalidad, en la actualidad, son las infecciones. 3. La sarcoidosis es una de las patologas que ms recurre. 4. Para conrmar el rechazo agudo el procedimiento de eleccin es la biopsia transbronquial. 5. El rechazo crnico es sinnimo de bronquiolitis obliterante y el principal factor de riesgo es la infeccin por CMV.

273.

Uno de los siguientes datos orienta el diagnstico diferencial entre el taponamiento cardaco y la pericarditis constrictiva a favor de la segunda. Seale cul: 1. Antecedente de pericarditis aguda. 2. Disminucin inspiratoria de la presin arterial sistlica de al menos 10 mmHg. 3. Ausencia de disminucin de la presin venosa yugular durante la inspiracin. 4. Bajo voltaje de los complejos QRS en el electrocardiograma. 5. Igualacin de las presiones diastlicas de ambos ventrculos en el cateterismo cardaco.

278.

Paciente de 32 aos con disnea leve. Analticamente muestra una anemia microctica e hipocrmica con creatinina de 1,8 mg/dl con alteraciones en el sedimento urinario. La Rx

-35-

EXAMEN ENARM 2/10

de trax muestra un patrn alveolar bilateral y tiene un patrn funcional en el rango de la normalidad, salvo una capacidad de difusin de monxido de carbono (DLCO) de 155%. De las siguientes determinaciones, cul NO es til para el proceso diagnstico? 284. 1. 2. 3. 4. 5. 279. Determinacin de niveles de c-ANCA. Anticuerpos antimembrana basal glomerular. Anticuerpos antinucleares (ANA). Niveles de inmunoglobulina. Niveles de protena C reactiva.

1. 2. 3. 4. 5.

T3 T2 T1 T4 T3

N1 N1 N2 N1 N0

M0. M0. M0. M0. M0.

El secuestro broncopulmonar se localiza preferentemente en: 1. 2. 3. 4. 5. Lbulo superior derecho. Segmento de la lngula. Segmento posterior del lbulo inferior. Lbulo superior izquierdo. Puede localizarse en todos los lbulos.

Varn de 56 aos, fumador de 15 cigarrillos/ da y obeso, que acude a consulta porque desde hace meses viene notando que se queda dormido en su trabajo durante el da y se nota irritable, aunque lo atribuye a las frecuentes cefaleas matutinas que padece y a sus frecuentes fallos de memoria, que le perjudican notablemente en su trabajo. La auscultacin cardiopulmonar es normal, aunque su tensin arterial es de 160/105 mmHg. En el estudio polisomnogrco se identican episodios de cese del ujo de aire por boca y nariz a pesar de las muestras de esfuerzo ventilatorio continuado. Cul es su diagnstico? 1. Sndrome de Ondina (hipoventilacin alveolar primaria). 2. Enfermedad pulmonar obstructiva crnica. 3. Apnea obstructiva del sueo. 4. Apnea central del sueo. 5. Cor pulmonale.

280.

Se encuentra usted durante un viaje de Mdicos sin fronteras en Nepal. Suponiendo que usted dispone de los medios adecuados, acude a la urgencia una de las voluntarias que presenta una PCO2 dentro de lmites normales y un gradiente alveolo-arterial de oxgeno marcadamente aumentado(50 mmHg). Si la tras la administracin de oxigenoterapia la PO2 no resulta corregible cul de los siguientes mecanismos subyacentes podra ser responsable de su cuadro? 1. Crisis de miastenia gravis. 2. Valvulopata mitral descompensada. 3. Disminucin de la presin atmosfrica de oxigeno. 4. Enfermedad pulmonar intersticial. 5. EPOC tipo ensema.

285.

Mujer de 58 aos diagnosticada de neumona bacteriana. En la radiografa de trax se aprecia derrame pleural loculado con afectacin del 20% del hemitrax derecho. Las caractersticas del lquido pleural son: aspecto purulento, pH 7,05; LDH 1520 UI/l y glucosa 25 mg/dl. Cul sera la actitud teraputica ms adecuada? 1. Actitud expectante y repetir toracocentesis a las 10 horas. 2. Tratamiento antibitico. 3. Antibioterapia + insercin de tubo de trax. 4. Antibioterapia + insercin de tubo de trax + tratamiento tromboltico intrapleural. 5. Decorticacin.

281.

Cul de las siguientes NO constituye una causa de disminucin del volumen espiratorio mximo en el primer segundo: 1. Obstruccin traqueal. 2. Alteraciones de la perfusin pulmonar. 3. Broncoconstriccin. 4. Prdida de la traccin radial de la va area. 5. Exceso de secrecciones bronquiales. Uno de los siguientes sndromes paraneoplsicos est producido por el carcinoma epidermoide: 1. 2. 3. 4. 5. Hipercalcemia e hipofosfatemia. Hiponatremia. Hipopotasemia. Sndrome de Eaton-Lambert. Osteoartropata hipertrca.

286.

En los pacientes afectos con artritis reumatoide, los derrames pleurales: 1. Menos de 1% desarrollan derrames pleurales. 2. Habitualmente los derrames pleurales son bilaterales. 3. Cuando desarrollan derrame pleural suele ser de gran tamao. 4. Son ms frecuentes en los varones que en las mujeres con artritis reumatoide. 5. Nunca son la primera expresin de la enfermedad.

282.

287.

283.

Uno de estos tumores pulmonares NO es susceptible de tratamiento quirrgico con intencin curativa:

Un nio de 8 aos mantiene una crisis asmtica desde hace 50 horas, con pobre respuesta a la medicacin broncodilatadora y tiene ebre de 38,7 C que motiv tratamiento con antibiticos. Cul de las siguientes situaciones de

-36-

EXAMEN ENARM 2/10

equilibrio cido-base sera la ms propia de su situacin? 1. 2. 3. 4. 5. 288. Normalidad. Alcalosis respiratoria pura. Acidosis respiratoria pura. Acidosis mixta o combinada. Alcalosis metablica pura.

291.

En un hombre joven, con dolor orbitario unilateral, agudo e intenso, que le despierta por la noche del sueo desde hace varias semanas, y al que se le asocia enrojecimiento conjuntival, lagrimeo y congestin nasal ipsilateral, el diagnstico ms probable es: 1. 2. 3. 4. 5. Linfoma orbitario. Pseudotumor orbitario. Cefalea histamnica (neuralgia migraosa). Migraa clsica. Arteritis de la temporal.

Cul de los siguientes es el tratamiento MENOS efectivo para la neuralgia del glosofarngeo? 1. Fenitona. 2. Amitriptilina. 3. Rizotoma percutnea a la altura del agujero yugular. 4. Neurotoma del nervio glosofarngeo en el bulbo. 5. Carbamacepina.

292.

Una de las siguientes entidades NO es complicacin inmediata de los traumatismos. Selela: 1. 2. 3. 4. 5. Hematoma epidural. Hematoma intraparenquimatoso. Hematoma subdural. Sndrome postconmocional. Crisis parciales motoras.

289.

Ante un paciente con un tumor cerebral e hidrocefalia por obstruccin del sistema ventricular, que comienza a presentar signos evidentes de herniacin transtentorial, cul de las siguientes debe ser la primera eleccin teraputica? 1. Dexametasona 30-60 mg/da para reducir edema cerebral. 2. Metilprednisolona 120-200 mg/da para disminuir la presin intracraneal. 3. Drenaje ventricular. 4. Radioterapia sobre la tumoracin. 5. Medicacin anticonvulsivante para prevenir convulsiones.

293.

Varn de 25 aos que presenta una infeccin respiratoria aparentemente banal. Quince das ms tarde acude al hospital por prdida de fuerza en extremidades, ms ostensible en las inferiores, de varios das de evolucin. El examen fsico demuestra ausencia de reejos. Qu exploracin realizara en primer lugar? 1. 2. 3. 4. 5. Electromiografa. Arteriografa cerebral. Puncin lumbar. EEG. TC cerebral.

290.

Mujer de 65 aos que consulta, a instancias de los familiares, por prdida de memoria. Los familiares reeren que desde hace dos aos le han notado, de forma progresiva, prdida de memoria para hechos recientes, apata y falta de inters por cosas que antes la ilusionaban. ltimamente haba tenido problemas con el manejo del dinero al ir de compras y en una ocasin se perdi en la calle. La paciente tena antecedentes de hipertensin arterial en tratamiento con IECA. Exploracin: vigil, mini-mental test 22/30, lenguaje empobrecido, grasping bilateral e hiperreexia generalizada. Pruebas complementarias: vitamina B12, hormonas tiroideas, hemograma y bioqumica de valores normales. EEG: enlentecimiento difuso. TAC de crneo: dilatacin de surcos y circunvoluciones corticales y, secundariamente, del sistema ventricular (atroa corticosubcortical). Cul es el diagnstico ms probable? 1. 2. 3. 4. 5. Demencia multiinfarto. Hidrocefalia a presin normal. Enfermedad de Alzheimer. Enfermedad de Pick. Demencia de etiologa metablica-carencial.

294.

Un lactante de 6 meses, sin antecedentes de inters y con desarrollo psicomotor normal, presenta salvas de contracciones bruscas en exin y extensin de la cabeza, tronco y extremidades que le aparecen al despertarse. En el EEG encontramos una alteracin bilateral con ondas lentas de alto voltaje caticas y asncronas (hipsarritmia). Cul es el frmaco con el que se podra iniciar el tratamiento en este tipo de epilepsia? 1. 2. 3. 4. 5. ACTH. Acido valproico. Etosuximida. Lamotrigina. Fenobarbital.

295.

Un nio de 4 aos presenta episodios de inicio y nal sbitos y escasos segundos de duracin caracterizados por desconexin del medio con cese de la actividad en curso. La exploracin interictal es normal, mientras que un registro EEG ictal demuestra un patrn punta-onda generalizado y simtrico a 3 Hz. Seale el diagnstico correcto:

-37-

EXAMEN ENARM 2/10

1. 2. 3. 4. 5. 296.

Crisis parciales complejas. Ausencias tpicas. Crisis parciales simples. Narcolepsia. Sndrome de Lennox-Gastaut.

2. T4 total normal, T4 libre aumentada, TSH suprimida. 3. T4 total normal, T4 libre normal, TSH suprimida. 4. T4 total baja, T4 libre baja, TSH aumentada. 5. T4 total normal, T4 libre normal, TSH aumentada. 301. Un varn de 42 aos que pesa 172 kg se somete a una intervencin quirrgica de derivacin gstrica para el tratamiento de su obesidad. Cul ser la principal complicacin precoz de esta ciruga? 1. 2. 3. 4. 5. 302. Poliartritis. Hepatopata progresiva. Saciedad precoz y vmitos. Diarrea. Nefrolitiasis.

Seale el enunciado FALSO en relacin a la esclerosis mltiple: 1. Las lesiones se distribuyen sobre todo a nivel periventricular. 2. La mayora de las lesiones son asintomticas. 3. Hay relacin directa entre el nmero de placas y la severidad clnica del paciente. 4. Es ms frecuente en mujeres. 5. Es ms frecuente en la raza blanca.

297.

La parlisis de la neurona motora inferior se caracteriza por todo lo siguiente, EXCEPTO: 1. 2. 3. 4. 5. Se pueden afectar grupos musculares reducidos. Atroa muscular intensa. Flaccidez e hipotona de los msculos afectados. Signo de Babinski. Fasciculaciones.

Cul de los siguientes perles corresponde a un pseudohipoparatiroidismo tipo II? (respuesta de AMPc nefrgeno a infusin de PTH: AMPc-PTH; respuesta del fsforo urinario a la infusin de PTH: Po-PTH): 1. PTH baja, AMPc-PTH aumenta, Po-PTH aumenta. 2. PTH alta, AMPc-PTH no aumenta, Po-PTH no aumenta. 3. PTH alta, AMPc-PTH aumenta, Po-PTH no aumenta. 4. PTH normal, AMPc-PTH aumenta, Po-PTH aumenta. 5. PTH baja, AMPc-PTH no aumenta, Po-PTH no aumenta.

298.

Una lesin del ncleo del tercer par craneal produce: 1. Lagoftalmos y desviacin del ojo hacia fuera y hacia abajo. 2. Ptosis palpebral y desviacin del ojo hacia dentro y arriba. 3. Lagoftalmos y desviacin del ojo hacia dentro y abajo. 4. Ptosis palpebral y desviacin del ojo hacia fuera y arriba. 5. Ptosis palpebral y desviacin del ojo hacia fuera y abajo.

303.

299.

Un enfermo acude a Urgencias con astenia, anorexia de meses de evolucin, nuseas, vmitos y dolor abdominal. Su TA es de 80/50 mmHg, con un potasio de 4 mg/ml y sodio de 128 mg/ ml. Lo primero que hay que hacer es: 1. Antiemticos para que deje de vomitar. 2. Extraccin de cortisol basal. 3. Instaurar tratamiento con hidrocortisona i.v. y udrocortisona oral. 4. Instaurar tratamiento con hidrocortisona i.v. 5. Instaurar tratamiento con hidrocortisona i.v. y uidoterapia intensiva. 304.

Una mujer de 66 aos consulta por la aparicin, de forma progresiva, de una tumoracin en la lnea media cervical, que relaciona en el tiempo con el traslado de su residencia a su pueblo en la sierra de Gredos cuando se jubil su marido. No reere ningn otro sntoma ni signo, excepto la sensacin de ocupacin en el cuello y una disfagia para slidos que se va agravando a medida que crece la masa. Cul sera su actitud diagnstica? 1. 2. 3. 4. 5. Laringoscopia indirecta. Endoscopia esofagogstrica. Radiografa de contraste con bario. Traqueobroncoscopia. Ecografa tiroidea.

300.

Una paciente de 65 aos acude a nuestra consulta con un informe mdico en el que consta el diagnstico de bocio multinodular con hipertiroidismo subclnico. Cul de los siguientes patrones hormonales le corresponde? 1. T4 total aumentada, T4 libre normal, TSH normal.

El diagnstico bioqumico del dcit de 21-hidroxilasa se conrma fundamentalmente por los niveles sricos elevados de: 1. 2. 3. 4. 5. Aldosterona. 11-desoxicortisol. 17-OH-progesterona. 11-desoxicorticosterona. Ninguno de los anteriores.

-38-

EXAMEN ENARM 2/10

305.

Cul es el frmaco de eleccin en el tratamiento de la diabetes inspida nefrgena? 1. 2. 3. 4. 5. Desmopresina. Vasopresina acuosa. Clorpropamida. Clobrato. Hidroclorotiacida.

3. LES. 4. Sndrome de Reiter. 5. Sndrome de Behet. 310. Una paciente de 60 aos de edad consulta por cefalea, prdida brusca de visin y claudicacin mandibular. A la exploracin presenta la arteria temporal derecha engrosada, dolorosa y sin pulso. Junto a este cuadro clnico, seale cul de los siguientes hallazgos analticos le impresiona de mayor especicidad diagnstica: 1. Anemia normoctica normocrmica. 2. Elevacin de la fosfatasa alcalina. 3. Elevacin de la velocidad de sedimentacin globular. 4. Hipergammaglobulinemia IgG. 5. Hipocomplementemia. 311. Varn de 35 aos consulta por una inamacin aguda en el tobillo izquierdo 2 das despus de una tremenda borrachera, durante la cual no recuerda haberse dado ningn golpe. El lquido obtenido por puncin es uido, turbio y acuoso con 13.450 leucos/mm3 y cristales en forma de aguja con intensa birrefringencia negativa. Cul es el diagnstico ms probable? 1. 2. 3. 4. 5. 312. Sinovitis villonodular pigmentada. Artropata alcohlica aguda. Artritis por cristales de oxalato clcico. Artritis por critales de pirofosfato clcico. Gota.

306.

Paciente de 70 aos acude porque desde hace varias semanas presenta astenia, decaimiento, dolores musculares sobre todo en piernas y brazos y depresin. En la exploracin destaca un aumento de la frecuencia cardaca. En ECG: brilacin auricular con respuesta ventricular rpida (140 lpm). Seale la actitud INCORRECTA: 1. Administrar betabloqueantes para controlar la frecuencia cardaca. 2. Iniciar tratamiento con antitiroideos. 3. Aadir yodo en forma de contrastes yodados o lugol. 4. Suministrar I-131. 5. Corticoides a dosis altas.

307.

En un paciente acromeglico NO esperara encontrar: 1. 2. 3. 4. 5. Apnea del sueo. Artritis reumatoide. Acantosis nigricans. Sndrome del tnel carpiano. Hipertensin arterial.

308.

Mujer de 50 aos con antecedentes de hepatitis crnica C que presenta cuadro de astenia, anorexia, poliartralgias, prpura palpable, Raynaud, acrocianosis y neuropata perifrica. En la analtica presenta VSG elevada, creatinina 2,5 mg/dl, factor reumatoide positivo y disminucin de las fracciones C3 y C4 del complemento. Los ANA y ANCA son negativos. Usted sospecha que la paciente presenta: 1. 2. 3. 4. 5. Panarteritis nodosa clsica. Policondritis recidivante. Granulomatosis de Wegener. Angeitis granulomatosa alrgica. Crioglobulinemia mixta esencial.

Uno de los siguientes hallazgos radiolgicos NO es sugestivo de ensema: 1. 2. 3. 4. 5. Diafragmas aplanados. Incremento de la transparencia retroesternal. ndice cardiotorcico aumentado. Incremento de la transparencia retrocardaca. Las sombras broncovasculares no alcanzan la periferia del pulmn.

313.

309.

Varn de 31 aos consulta por episodios repetidos de artritis inamatoria oligoarticular asimtrica de medianas articulaciones desde hace tres aos, cuya duracin es de 2-4 semanas. Actualmente presenta unas lceras de aspecto sucio en la boca, tiene ebre, ligero dolor abdominal, cefalea intensa y signos de tromboebitis en la pantorrilla derecha. Cul es el diagnstico ms probable? 1. Colitis ulcerosa. 2. Enfermedad de Whipple.

Un paciente varn, drogadicto por va parenteral y en tratamiento con zidovudina, 3 TC y saquinavir ingresa con cuadro febril de origen desconocido. Los hemocultivos incluidos los de larga incubacin, son negativos. Es instaurada antibioterapia emprica de amplio espectro y nalmente anfotericina B intravenosa por sospecha de inmunodepresin. Como complicacin presenta monoartritis sptica de rodilla. Seale la opcin INCORRECTA: 1. El diagnstico probable es artritis candidisica que junto con la blastomictica, es de rpida progresin. En el resto de las artritis micticas el diagnstico puede demorarse meses. 2. En caso de histoplasmosis se observa una poliartritis migratoria con eritema nudoso.

-39-

EXAMEN ENARM 2/10

3. En la blastomicosis es frecuente la manifestacin articular aislada. Rara vez en el contexto de un sndrome pulmonar-cutneo-artrtico. 4. En caso de extensa formacin de pannus granulomatoso articular es til el tratamiento quirrgico. 5. En caso de esporotricosis, el agente etiolgico (Sporothix schenkii) se cultiva fcilmente en el lquido sinovial. 314. Un paciente de cincuenta aos con AR en tratamiento con AINE presenta dolores moderados en muecas y rodillas. En la radiografa de ambas manos hay osteoporosis yuxtaarticular y erosiones en tercera, cuarta y quinta articulaciones metacarpofalngicas de ambas manos. Cul es el tratamiento de eleccin? 1. Aumentar la dosis de los AINE. 2. Mantener la misma dosis de AINE y revisar en un mes. 3. Suspender los AINE y dar prednisona. 4. Mantener los AINE y aadir un frmacos modicador de la enfermedad 5. Aadir iniximab. 315. En un paciente con esclerodermia cutnea difusa (esclerosis sistmica) que ha permanecido estable durante varios aos, se observa una hipertensin de reciente comienzo. Este paciente tiene un riesgo signicativo de fallecer por: 1. 2. 3. 4. 5. 316. Ictus trombtico. Hemorragia en el sistema nervioso central. Insuciencia renal. Hipertensin pulmonar. Fibrosis pulmonar.

4. Lesin del nervio mediano. 5. Lesin de la arteria radial. 319. Seale la opcin INCORRECTA en relacin al tratamiento de una lesin en la rodilla por un golpe con la articulacin en hiperextensin forzada: 1. El tratamiento conservador en pacientes jvenes, hace muy probable el desarrollo de artrosis y lesiones meniscales secundarias. 2. El tratamiento conservador incluye un periodo de 4 semanas de inmovilizacin con frula y descarga. 3. Para la reconstruccin quirrgica, es de eleccin el autoinjerto tendinoso. 4. Cuando hay lesin concomitante de los ligamentos colaterales, su reparacin debe diferirse unos meses y realizarse en otra intervencin. 5. La rehabilitacin debe incluir ejercicios isomtricos para el cudriceps y los isquiotibiales. 320. En la enfermedad de Legg-Calv-Perthes pueden denirse 5 fases radiolgicas dentro de lo que es la evolucin de la propia enfermedad. Las siguientes respuestas corresponden a cada una de ellas EXCEPTO una, que es consecuencia tarda de las deformidades residuales; selela: 1. Disminucin de tamao del ncleo de crecimiento afectado en relacin al contralateral. 2. Artrosis lo sucientemente avanzada como para requerir artroplastia total de cadera. 3. Remodelacin de la epsis proximal del fmur y del acetbulo. 4. reas de densidad aumentada junto a zonas de intensa radiotransparencia en la epsis femoral. 5. Pseudoquistes metasarios y trastornos en la zona anterolateral del cuello. 321. Sobre la luxacin congnita de cadera, cul de las siguientes armaciones es FALSA? 1. 2. 3. 4. 5. Es ms frecuente en nias. Es ms frecuente en primognitos. Es ms frecuente en partos de nalgas. Suele ser posterior. El tratamiento debe comenzarse pasadas dos semanas, una vez iniciada la osicacin de la cabeza femoral.

Son factores favorecedores de la formacin de callo seo todos los siguientes, EXCEPTO: 1. 2. 3. 4. 5. Glucocorticoides Hormona de crecimiento. Hormona tiroidea. Agentes anabolizantes. Vitamina D.

317.

El tratamiento de una fractura no desplazada de tercio proximal de hmero ser: 1. Inmovilizacin con Velpeau. 2. Inmovilizacin con vendaje en ocho de guarismo. 3. Yeso colgante. 4. Fijacin interna. 5. Sustitucin protsica.

322.

Dentro del tratamiento no farmacolgico de la HTA lbil se incluyen las siguientes medidas dietticas, EXCEPTO: 1. 2. 3. 4. 5. Dieta sin sal. Reduccin de peso. Ejercicio fsico isomtrico. Suspender el tabaco. Suspender la toma de anticonceptivos orales.

318.

Qu lesin se asocia a la luxacin del semilunar? 1. Lesin de la arteria cubital. 2. Lesin del nervio cubital. 3. Lesin del nervio radial.

-40-

EXAMEN ENARM 2/10

323.

Cul es la cantidad mxima de orina diluida que puede eliminar un rin normal, antes de que se produzca hiponatremia? 1. 2. 3. 4. 5. 5 l. 10 l. 20 l. 30 l. 4 l.

poco, realizamos otras radiografas en las que encontramos reabsorcin peristica en falanges y huesos largos, formacin de quistes seos pardos y crneo en sal y pimienta. Aunque usted ya sospecha que el paciente padece osteodistroa renal, qu tipo especco de lesin sufre? 1. 2. 3. 4. 5. 327. Ostetis brosa qustica. Osteosclerosis. Raquitismo renal. Osteomalacia. Calcicaciones metastsicas.

324.

Una mujer de 58 aos presenta un cuadro de ebre de cuatro semanas, llegndose al diagnstico de toxoplasmosis. En las tres semanas previas haba recibido ampicilina oral a la dosis de 2 g/da, con lo que haba descendido algo, sin desaparecer la ebre. Unos das antes del ingreso vuelve a elevarse la ebre, aparece eritema, oliguria de 500 a 700 ml/da, creatinina plasmtica de 4,3 mg/dl y hematuria macroscpica con proteinuria de 2 g/24 h. En el sedimento el 80% de los hemates estn deformados y hay algunos eosinlos. Los niveles de C3 y C4 en plasma son normales. El cuadro revirti al nal, dejando una ltracin glomerular del 75% Qu diagnstico, de los siguientes, es el ms probable? 1. 2. 3. 4. Nefritis intersticial aguda por hipersensibilidad. Glomerulonefritis aguda postinfecciosa. Necrosis tubular aguda. Glomerulonefritis mesangiocapilar tipo II (depsitos densos). 5. Glomerulonefritis membranosa.

Cul de las siguientes opciones utilizara para hacer el diagnstico de certeza de ateroembolia renal? 1. Visualizacin de mbolos de colesterol en el fondo de ojo. 2. El diagnstico es clnico y por los datos de laboratorio. 3. Demostrando una fuente potencial de micrombolos de colesterol, en un paciente con factores de riesgo. 4. Observando cristales de colesterol en pequeas arterias y arteriolas de la biopsia del msculo esqueltico o piel asintomticos. 5. El diagnstico de certeza slo puede hacerse en el anlisis anatomopatolgico de la necropsia.

328.

325.

En una mujer de 45 aos que aqueja cefaleas, anorexia y edemas, el clnico descubre de primera intencin una hipertensin arterial, dos riones palpables, abollonados y una azotemia. La mujer recuerda que su madre, fallecida hace tiempo, tambin padeci del rin. Aunque el clnico tiene ya sucientes datos para sospechar el diagnstico, quiere conrmarlo y decide solicitar una prueba denitiva, cul? 1. Un cariotipo porque cree que es un rin multiqustico bilateral. 2. Una ecografa abdominal porque cree que es una poliquistosis renal. 3. Una cistoscopia para descartar una uropata obstructiva baja productora de hidronefrosis. 4. Una pielografa retrgrada bilateral para descartar una megacaliosis. 5. Una estudio urodinmico de aparato urinario superior porque cree que es una hidronefrosis por disfuncin de la unin pieloureteral.

Varn de 50 aos, hipertenso, en tratamiento con enalapril desde hace 5 aos. Nefrectoma izquierda secundaria a litiasis. Acude a Urgencias reriendo los siguientes antecedentes: 2 semanas antes present clico renal expulsivo sin precisar medicacin. Durante los das siguientes la diuresis oscil entre 4 litros durante varios das y posteriormente, 1 litro. 24 horas antes de la consulta: oliguria severa. A la exploracin se encuentra normohidratado, sin globo vesical ni otros hallazgos llamativos. En el sedimento de orina existe microhematuria y leucocitos aislados. Proteinuria negativa. El diagnstico ms probable es: 1. Insuciencia renal secundaria a nefroangioesclerosis. 2. Glomerulonefritis aguda. 3. Proceso renal obstructivo. 4. Insuciencia renal secundaria a hiperltracin. 5. Insuciencia renal secundaria a inhibidores de la enzima de conversin de angiotensina.

329.

Paciente en tratamiento reciente con hemodilisis que desarrolla de forma aguda: cefalea, nuseas, agitacin, somnolencia y convulsiones. Cul es el diagnstico ms probable? 1. 2. 3. 4. 5. Encefalopata por dilisis. Encefalopata urmica. Sndrome de desequilibrio. Linfoma cerebral primario. Mielinolisis central pontina.

326.

Paciente de 58 aos con insuciencia renal terminal en espera de la realizacin de un trasplante, acude a Urgencias porque se ha cado al notar que su pierna se quebraba. En la exploracin radiolgica encontramos una fractura espontnea de disis femoral, pero indagando un

-41-

EXAMEN ENARM 2/10

330.

Mujer de 50 aos, diabtica insulinodependiente, con infecciones urinarias y clicos nefrticos de repeticin. Acude a Urgencias por dolor en fosa renal izquierda de cinco das de evolucin, asociado en las ltimas 24 horas a ebre, escalofros y malestar general. Analtica de sangre: plaquetopenia, leucocitosis y disminucin de la actividad de la protrombina. Analtica de orina normal. Radiografa de abdomen con claras imgenes de litiasis. Eco renal: dilatacin moderada de sistema excretor izquierdo. Cul es la conducta ms adecuada? 1. Solicitar hemocultivos y urocultivo para establecer la necesidad de antibioterapia. 2. Realizar urografa intravenosa para intentar liar la causa. 3. Hidratar a la paciente bajo observacin rigurosa y repetir ecografa a las 48 horas. 4. Colocar catter doble-J o practicar nefrostoma percutnea de forma inmediata con cobertura antibitica. 5. Laparotoma exploradora para objetivar causa y realizar tratamiento antes de que el cuadro est muy evolucionado.

334.

NO es tpico de la leucemia linftica crnica: 1. Presencia de inmunoglobulinas en la membrana linfocitaria. 2. Hipergammaglobulinemia. 3. Anemia por anticuerpos calientes. 4. Ausencia de sntomas en estadios iniciales. 5. Trombopenia autoinmune.

335.

Cul es la causa de la mayor parte de los casos de crisis aplsicas transitorias que aparecen de forma brusca en los pacientes con enfermedad hemoltica crnica? 1. 2. 3. 4. 5. Infeccin bacteriana aguda. Enfermedades autoinmunes. Parvovirus B19. Toxicidad farmacolgica. Papilomavirus.

336.

La prueba ms fehaciente para establecer el diagnstico del dcit del factor XIII es: 1. Solubilidad del cogulo en urea o en cido monoactico. 2. Dosicacin del bringeno. 3. Tiempo de cefalina. 4. Tiempo de protrombina. 5. Dosicacin de los productos de degradacin del bringeno/brina.

331.

Ante un paciente de 24 aos que presenta ebre alta con dolor, inamacin y enrojecimiento testicular izquierdo, cul de las siguientes armaciones es INCORRECTA? 1. El diagnstico ms probable es el de epididimitis. 2. Los patgenos ms frecuentes son Chlamydia trachomatis y Neisseria gonorrhoeae. 3. El tratamiento de eleccin es vancomicina + gentamicina. 4. El tratamiento de eleccin puede ser ooxacino. 5. Un tratamiento alternativo es ceftriaxona en monodosis ms 10 das de doxiciclina. 337.

Un paciente varn de 35 aos de edad es diagnosticado de linfoma no Hodgkin linfoblstico estadio IV de la clasicacin de Ann Arbor, con sntomas B De qu factor depende fundamentalmente el tratamiento y el pronstico en este paciente? 1. 2. 3. 4. 5. Edad. Estadio Ann Arbor. Aparicin de sntomas B. Caractersticas de las adenopatas. Subtipo histolgico.

332.

En un paciente varn de 45 aos que presenta urgencia urinaria y tenesmo, con un PSA de 8 ng/ml y un tacto rectal que pone de maniesto una prstata de supercies irregulares y consistencia ptrea, qu estara recomendado realizar? 1. 2. 3. 4. 5. RM. TC. ECO prosttica transrectal y biopsia sextante. Radiografa simple de abdomen. UIV.

338.

Varn de 33 aos diagnosticado de linfoma de Hodgkin tras la biopsia de una linfadenopata laterocervical. En el estudio de extensin solamente se objetivan mltiples adenopatas en cadenas paraarticas e inguinales a nivel abdominal. En qu estadio de la clasicacin de Ann Arbor se encuentra este paciente? 1. 2. 3. 4. 5. Estadio Estadio Estadio Estadio Estadio I. II. III. IV. IIE.

333.

Seale la pareja INCORRECTA tratamiento - efecto secundario en la hiperplasia benigna prosttica: 1. 2. 3. 4. 5. Finasteride - alopecia. Alfa bloqueantes - hipotensin ortosttica. RTU - eyaculacin retrgrada. Alfa bloqueantes - eyaculacin retrgrada. Finasteride - prdida de la libido.

339.

En un paciente mayor se descubre accidentalmente en una radiografa de crneo una lesin en sacabocados aislada. Se le realiza una anamnesis y descubrimos que tiene dolor de espalda

-42-

EXAMEN ENARM 2/10

constante, que no empeora por la noche y que se exacerba con el movimiento. Presenta anemia y tuvo una pielonefritis recidivante haca un ao; cul sera su actuacin? 1. Realizar una serie radiogrca de la columna vertebral y torcica. 2. Administrar AINE sistmicos y reposo. 3. Realizar una gammagrafa para valorar la extensin de las lesiones. 4. Se diagnostica como granuloma eosinlo y se trata con radioterapia local. 5. Curetaje de la lesin craneal. 340. Varn de 2 aos de edad con historia de gingivorragias con la denticin y equmosis tras punciones y traumatismos mnimos que presenta hemorragia importante en el msculo glteo izquierdo tras la administracin de una inyeccin intramuscular. El hemograma es normal y en el estudio bsico de coagulacin presenta solamente un tiempo de tromboplastina parcial activada prolongado. Seale la FALSA respecto a este paciente: 1. Est contraindicada la administracin de AINE. 2. Debe realizarse una dosicacin de los factores de la va intrnseca de la coagulacin. 3. La ausencia de historia familiar hemorrgica descarta el diagnstico de hemolia. 4. Puede presentar como complicacin una parlisis nerviosa. 5. Las contracturas y atroa muscular son posibles secuelas. 341. Una mujer de 62 aos, sin antecedentes de inters, acude a Urgencias por hematomas espontneos en ambos miembros inferiores, epistaxis y petequias. La exploracin no pone nada ms de maniesto; el pulso es de 100 latidos/min y la TA de 180/110. Analticamente presenta hemoglobina de 13 g/dl, 6800 leucocitos (frmula normal) y 2000 plaquetas. El frotis no revela ninguna alteracin. La actitud ms correcta en este caso sera: 1. Ingreso de la paciente, control de la TA, e inicio de tratamiento con esteroides y gammaglobulina intravenosa. 2. Plasmafresis repetidas. 3. Alta, remitindola a consulta de forma preferente para estudio de mdula sea, sin poner tratamiento para no enmascarar el cuadro. 4. Ingreso hospitalario y transfusin inmediata de plaquetas. 5. Esplenectoma de urgencia. 342. Paciente de 17 aos, diagnosticado hace un ao de leucemia linfoide aguda en tratamiento de mantenimiento con 6-mercaptopurina y metotrexato. Acude a Urgencias por cefalea, nuseas y parlisis del III par craneal, por lo que usted

sospecha una recidiva de su leucemia. Qu prueba de las siguientes le parece ms razonable realizar para conrmar el diagnstico en este paciente? 1. 2. 3. 4. 5. 343. TC craneal. RM craneal. Puncin lumbar. Ecografa ocular. Frotis de sangre perifrica.

El tratamiento de eleccin en los sndromes trombticos microangiopticos consiste en: 1. 2. 3. 4. 5. Glucocorticoides. Esplenectoma. Antiagregantes plaquetarios. Trasplante de mdula sea. Plasmafresis.

344.

El botulismo es una enfermedad paralizante desencadenada por potentes neurotoxinas producidas por el Clostridium botulinum, que pueden ocasionar la muerte del paciente. Seale cul de las siguientes opciones caracteriza al cuadro neurolgico de los pacientes afectados: 1. Parlisis ascendente junto con alteraciones sensitivas. 2. Parlisis descendente simtrica con manifestaciones sensoriales. 3. Parlisis descendente asimtrica con manifestaciones sensoriales. 4. Parlisis generalizada de instauracin fulminante con predominio en miembros inferiores. 5. Parlisis aislada de msculos extensores de miembros, con la instauracin caracterstica de temblor intencional.

345.

Seale cul es el tratamiento emprico de la infeccin de herida producida por mordedura de gato de ms de doce horas de evolucin: 1. 2. 3. 4. 5. Doxiciclina. Amoxicilina-clavulnico. Ciprooxacino. Claritromicina. Gentamicina.

346.

En un paciente con un cuadro de diarrea disenteriforme severa, el tratamiento antibitico emprico de eleccin sera: 1. 2. 3. 4. 5. Ampicilina. Amoxicilina. Cefalosporina de 3 generacin. Cotrimoxazol. Ciprooxacino.

347.

Varn de 49 aos que consulta por disuria y hematuria marcada. En la exploracin fsica no se objetiva ningn dato de inters. El paciente

-43-

EXAMEN ENARM 2/10

est afebril. En la radiografa de trax se aprecian lesiones apicales residuales no sugestivas de actividad, y en una urografa intravenosa se aprecia hidronefrosis con focos de calcicacin y distorsin del urter derecho. El sedimento de orina muestra hematuria y leucocituria. Se enva muestra para cultivo y avisan del laboratorio de microbiologa que ha crecido en 24 horas un microorganismo que est pendiente de identicacin. Cul es el diagnstico ms probable? 1. 2. 3. 4. 5. 348. Pielonefritis por enterococo. Pielonefritis por E. coli. Tuberculosis genitourinaria. Nefrolitiasis. Cistitis bacteriana.

351.

Seale en cul de las siguientes infecciones, la penicilina sigue siendo el tratamiento de eleccin a pesar de las resistencias bacterianas y de la sntesis de mltiples antimicrobianos nuevos: 1. 2. 3. 4. 5. Meningitis por Neisseria meningitidis B. Otitis por Moraxella catarrhalis. Neumona por Streptococcus pneumoniae. Absceso pulmonar por Peptostreptococcus. Uretritis por Neisseria gonorrhoeae.

352.

La fasciola heptica NO produce: 1. Eosinolia. 2. Hepatomegalia. 3. Incremento de incidencia de colangiocarcinoma. 4. Urticaria. 5. Colelitiasis.

Paciente de 69 aos, jubilado, que en su tiempo libre se dedica a cuidar, en una granja escuela, un grupo de asnos. Tiene diversos ndulos en el brazo derecho, algunos de los cuales han evolucionado a lceras, de las cuales surgen unos trayectos eritematosos que usted identica como linfangitis. En la exploracin encuentra adenopatas axilares y cervicales. Tiene dicultad respiratoria; reere odinofagia y cambio en la textura de su voz desde hace unos das, que atribuye usted a la afectacin de las mucosas orofarngea y nasal. La piel del dorso de la nariz tiene aspecto erisipeloide. Usted ya sospecha de la siguiente etiologa: 1. 2. 3. 4. 5. Burkholderia mallei. Burkholderia pseudomallei. Chryseobacterium indologenes. Pseudomona aeruginosa. Alcaligenes xylosoxidans.

353.

Un hombre de 50 aos consult por ebre prolongada que le haba comenzado hace dos semanas. La ebre era de 39 C, tpicamente nocturna y, a veces, con dos ascensos diarios. Tambin presenta cefalea, malestar y dolores articulares. A la exploracin fsica se palpa esplenomegalia por debajo del reborde costal. La analtica muestra: Hb 10 g/dl, 80.000 plaquetas, 3.000 leucocitos/mm3, albumina 3 g/dl, gammaglobulina 4 g/dl. Cul es el diagnstico que le sugiere este cuadro? 1. 2. 3. 4. 5. Leishmaniasis. Brucellosis. Fiebre Q. Fiebre botonosa mediterrnea. Histoplasmosis.

354. 349. Ante un paciente con una endocarditis infecciosa por Staphylococcus aureus meticilin-resistente, qu tratamiento aconsejara? 1. 2. 3. 4. 5. 350. Cloxacilina. Cefotaxima. Imipenem. Aztreonam. Vancomicina.

Un varn de 30 aos procedente de Guatemala instaura un cuadro que incluye prurito, ndulos subcutneos en cabeza y cuello, conjuntivitis, queratitis y astenia. De entre los siguientes agentes etiolgicos, le parece ms probable como causante de esta clnica: 1. 2. 3. 4. 5. Onchocerca volvulus. Loa-Loa. Schistosoma mansoni. Wuchereria bancrofti. Mansonella perstans.

Respecto a la ebre de origen desconocido, una respuesta es FALSA: 355. 1. La historia clnica constituye la base del diagnstico. 2. La mdula sea es un rgano rentable para una biopsia a ciegas. 3. En la biopsia heptica de la ebre por frmacos pueden objetivarse granulomas. 4. Los hematomas retroperitoneales pueden ser una causa de ebre de origen desconocido. 5. En pacientes ancianos el pronstico suele ser mejor.

Una mujer de 28 aos, ex ADVP, es diagnosticada de infeccin por VIH a raz de presentar una candidiasis orofarngea. Actualmente se encuentra asintomtica y su cifra de linfocitos CD4 es de 180/mm3. Qu actitud tomara usted en este momento? 1. Seguimiento clnico y analtico sin tratamiento, ya que la paciente se encuentra asintomtica. 2. Seguimiento clnico y prolaxis con uconazol para prevenir recurrencias de la candidiasis.

-44-

EXAMEN ENARM 2/10

3. Dado que los linfocitos CD4 son inferiores a 500/mm3, tratar con dos inhibidores de transcriptasa inversa. 4. Solicitar una determinacin de carga viral y, si es superior a 30.000 copias de ARN/ml, iniciar tratamiento con dos inhibidores de transcriptasa inversa. 5. Iniciar tratamiento con dos inhibidores de transcriptasa inversa y un inhibidor de proteasa. 356. El mecanismo de accin principal de los antiinamatorios no esteroideos (AINE) es: 1. La inhibicin de la ciclooxigenasa y, en consecuencia, de la biosntesis de prostaglandinas a nivel tisular. 2. El aumento de la sntesis de prostaglandinas. 3. La inhibicin de la transmisin perifrica (bras A-delta) del estmulo nervioso. 4. Desconocido, a nivel del sistema nervioso. 5. El aumento del umbral de percepcin de estmulos dolorosos. 357. Cul de los siguientes efectos NO es tpico de los opiceos: 1. 2. 3. 4. 5. 358. Supresin de la tos. Nuseas y vmitos. Estreimiento. Disminucin del tono del esfnter vesical. Aumento de la presin intracraneal.

1. 2. 3. 4. 5. 362.

Maculopata txica. Degeneracin macular senil. Coriorretinopata serosa central. Estras angioides. Edema macular cistoide.

Una nia, a los 15 meses, es remitida al oftalmlogo por estrabismo convergente y leucocoria del ojo derecho. Cul es el primer diagnstico diferencial a realizar? 1. 2. 3. 4. 5. Catarata congnita por rubola. Enfermedad de Coats. Parlisis de los pares craneales IV y VI. Retinoblastoma. Desprendimiento de retina.

363.

Un paciente es diagnosticado de artritis reactiva a las 2 semanas de presentar una infeccin entrica. Presenta afectacin de las articulaciones de ambos pies y derrame sinovial en rodilla derecha. De los siguientes microorganismos involucrados habitualmente en la etiopatognica del proceso seale, por favor, el ms frecuente: 1. 2. 3. 4. 5. Shigella exneri. Yersinia enterocoltica. Yersinia pseudotuberculosis. Campylobacter yeyuni. Chlamydia trachomatis.

364. Varn de 29 aos con dolor abdominal recidivante y angioedema de labios, lengua y laringe, qu patrn de complemento sera ms probable en este paciente? 1. 2. 3. 4. 5. 359. Dcit de C2. Dcit de C3. Dcit de C8 Ausencia de C1 inhibidor. Dcit de C3 y factor B con CH50 disminuido. 365.

Varn de 70 aos que acude por hipoacusia. En la exploracin tiene un Rinne negativo en OD y positivo en OI y un Weber lateralizado hacia el OI. Qu tipo de hipoacusia tiene el paciente? 1. 2. 3. 4. 5. Cofosis de Hipoacusia Hipoacusia Hipoacusia Hipoacusia OD. de transmisin de OD. de percepcin de OI. mixta de OD. de transmisin de OI.

Cul es el agente patgeno de la tia favosa? 1. 2. 3. 4. 5. Tricophiton mentagrophites. Tricophiton scholeini. Malassezia furfur. Microsporum audouini. Epidermotum inguinale.

El sntoma ms frecuente de los cnceres de glotis es: 1. Disfona. 2. Disfagia. 3. Tos. 4. Disnea. 5. Prdida de peso. En un paciente con una parlisis larngea izquierda con la cuerda vocal en posicin intermedia, disfagia y aspiracin, sensibilidad farngea conservada y nistagmo vertical, la lesin causal se localiza en: 1. Isquemia bulbar por obstruccin de la arteria cerebelosa posteroinferior (sndrome de Wallemberg). 2. Cncer de cavum con extensin a nivel del agujero rasgado posterior.

360.

NO es una lesin caracterstica de la retinopata diabtica: 1. 2. 3. 4. 5. Exudados algodonosos. Microaneurismas. Estrechamiento arterial generalizado. Edema macular. Neovascularizacin retiniana.

366.

361.

Seale la causa ms frecuente de degeneracin macular:

-45-

EXAMEN ENARM 2/10

3. La corteza cerebral. 4. Cncer de tiroides con lesin del nervio larngeo recurrente. 5. Tiroidectoma antigua con lesin del nervio larngeo superior. 367. Ante una paciente diagnosticada de trastorno bipolar, tratada con litio desde hace aos, con un buen control de la enfermedad y que es diagnosticada de nefropata intersticial con una disminucin grave del aclaramiento renal, cul sera la actitud ms adecuada? 1. Bajar dosis de litio. 2. Retirar litio y sustituirlo por un neurolptico. 3. Seguir el tratamiento con litio, puesto que la retirada podra producir una descompensacin de la enfermedad. 4. Retirar litio y sustituirlo por carbamacepina. 5. Aadir un diurtico tiacdico para poder disminuir dosis de litio.

370.

La necesidad de aumentar la dosis de una droga para conseguir el efecto deseado, o la disminucin del efecto al mantener la misma dosis se denomina: 1. 2. 3. 4. 5. Tolerancia. Dependencia. Abstinencia. Refuerzo positivo de la sustancia. Craving.

371.

Cul de los siguientes es el trastorno endocrinolgico que con mayor frecuencia produce demencia? 1. 2. 3. 4. 5. Hipotiroidismo. Dcit de vitamina B12. Dcit de tiamina. Hipoglucemia crnica. Dcit de cido nicotnico.

372. 368. Varn de 36 aos que es trado por su madre porque le tiene miedo. sta es su dcima hospitalizacin, la primera fue con 15 aos, y desde entonces nunca ha vuelto al colegio ni ha tenido un trabajo. Su afecto va desde la ira hacia su madre (me da mierda para comer, lo que sale del recto de otras personas) hasta las risitas tontas hacia el mdico. Su madre cuenta que hace un mes dej la medicacin y comenz a or voces y a actuar con mucha extravagancia. Su lenguaje es disgregado, con rimas y asociaciones laxas. Dice he estado comiendo alambres y encendiendo fuegos. El paciente no tiene historia conocida de abuso de alcohol o drogas. El diagnstico ms probable es: 1. 2. 3. 4. 5. 369. Esquizofrenia hebefrnica. Esquizofrenia paranoide. Trastorno delirante crnico. Demencia. Trastorno esquizotpico.

Paciente varn de 25 aos de edad, soltero y de nivel socioeconmico alto. Acude a consulta de psiquiatra por un cuadro caracterizado por: emplear horas antes de acostarse en comprobar que ha cerrado la espita del gas y las luces de casa, considera que los objetos de su escritorio deben estar en una determinada posicin. Para aliviar esta necesidad de tener todo en orden y como quiere, tamborilea diez veces sobre una mesa de madera y despus se queda tranquilo. Tiene un estado anmico triste y empieza a sentirse desbordado por la situacin. Cul sera su diagnstico? 1. 2. 3. 4. 5. Tumor frontal. Depresin neurtica. Neurosis obsesiva. Esquizofrenia simple. Neurosis fbica.

373.

El tratamiento que ofrece mayores garantas de xito en la anorexia nerviosa es: 1. 2. 3. 4. 5. Psicoterapia cognitivo-conductual. Psicoterapia familiar. Tratamiento antidepresivo. Frmacos antianorexgenos (ciproheptadina). Hormonoterapia, para fomentar el crecimiento, inducir la menstruacin y corregir desequilibrios hormonales.

Acude a Urgencias un paciente con sndrome de abstinencia alcohlica, administrndole 50 mg de cloracepato i.m. A los 30 minutos el tratamiento se demuestra insuciente y decide dar otra dosis, que debe volver a repetir otra vez ms, hasta hacerse efectivo. Varias horas despus, el paciente est atxico y estuporoso, a qu se debe esta nueva situacin? 1. A la prolongada vida media del cloracepato. 2. A la absorcin errtica y retardada del cloracepato. 3. Al efecto sumativo del alcohol y el cloracepato. 4. A la inecacia del cloracepato en el sndrome de abstinencia alcohlica. 5. A un efecto idiosincrtico tpico del cloracepato.

374.

El tratamiento ms til para la intoxicacin por organofosforados es: 1. 2. 3. 4. 5. Oximas. Atropina. Acetilcolina. Betanecol. Bromuro de ipatropio.

375.

Paciente de 26 aos, nulpara, con amenorrea de 3 meses acude por hemorragia vaginal

-46-

EXAMEN ENARM 2/10

moderada. En la exploracin se objetiva un tero mayor del que correspondera al tiempo de amenorrea. La HCG es fuertemente positiva y la ecografa muestra una imagen semejante a los copos de nieve en la cavidad uterina. Qu es FALSO en relacin a esta patologa? 1. Puede ser causa de preeclampsia antes de la 20 semana. 2. Puede cursar con clnica de tirotoxicosis. 3. Puede coexistir con quistes tecalutenicos en el ovario. 4. El tratamiento de eleccin es la histerectoma. 5. La determinacin seriada de HCG es til para su seguimiento. 376. Mujer con dolores de parto a las 38 semanas de gestacin y sangrado moderado de color rojo intenso. El latido fetal se ausculta fcilmente y est dentro de la normalidad. El diagnstico es de placenta previa que ocluye totalmente el cuello del tero. Cul es la actitud a seguir ms correcta? 1. 2. 3. 4. 5. Actitud expectante. Parto vaginal dejado a su evolucin. Cesrea. Parto con forceps. Amniorrexis y goteo de oxitcicos para terminar el parto lo ms rapidamente posible.

tes, recomendara porque NO se asocia a la produccin de este cuadro? 1. 2. 3. 4. 5. 379. Trigo. Cebada. Arroz. Centeno. Avena.

Paciente de 28 aos que acude al gineclogo por prurito vulvovaginal y leucorrea. A la exploracin existe abundante ujo amarillogrisceo espumoso y la vagina y el exocrvix muestran un enrojecimiento difuso y un punteado ms intenso. Cul sera el tratamiento ms adecuado en este caso? 1. 2. 3. 4. 5. Amoxicilina y clavulnico. Clotrimazol. Estrgenos tpicos. Doxiciclina. Metronidazol.

380.

Un lactante de 6 meses que, estando previamente bien, comienza bruscamente con vmitos asociados a episodios de llanto intenso, palidez y heces en jarabe de grosella, presenta como diagnstico ms probable: 1. 2. 3. 4. 5. Gastroenteritis vrica. Apendicitis aguda. Clicos del lactante. Invaginacin intestinal. Diarrea secretora.

377.

Mujer de 20 aos que acude a su consulta reriendo menstruaciones irregulares que incluyen baches amenorreicos. No relaciones sexuales. Antecedentes personales: enfermedades propias de la infancia; amigdalectoma; no alergias medicamentosas conocidas. Exploracin: en el tacto rectal parece palparse un aparato genital normal; resto de exploracin fsica sin datos de inters salvo talla de 162 cm y peso de 75 kg. Se realiza ecografa que muestra un tero regular de 60 mm y unos anejos ligeramente aumentados de tamao con mltiples imgenes econegativas compatibles con folculos, la mayor de los cules tiene 9 mm de dimetro. Seale la respuesta INCORRECTA: 1. Lo ms probable es que encontremos en la analtica un cociente LH/FSH mayor de 2. 2. El tratamiento de eleccin de esta paciente se basa en la administracin de antiandrgenos tipo acetato de ciproterona. 3. Esta paciente mejorara su cuadro clnico con una simple reduccin del IMC. 4. Los niveles de testosterona total pueden oscilar en torno a 1 ng/ ml. 5. Algunas pacientes pueden presentar Acantosis nigricans.

381.

Un mioma producir con mayor frecuencia: 1. Sangrado abundante en el momento esperado de la regla (hipermenorrea). 2. Hemorragia acclica (metrorragia). 3. Reglas muy frecuentes (polimenorrea). 4. Retrasos menstruales: ciclos muy largos (opsomenorrea). 5. Flujo vaginal serosanguinolento.

382.

Una paciente primigesta sin antecedentes de inters y con controles analticos previos normales, presenta glucemias basales de 148 y 152 mg/dl en dos determinaciones realizadas en la 22 semana de gestacin. Cul es la actitud ms correcta en este caso? 1. Realizar una prueba de tolerancia a la glucosa con sobrecarga oral de 100 g. 2. Realizar una prueba con sobrecarga oral de glucosa de 75 g y, en caso de normalidad, repetirla con 100 g. 3. Realizar una prueba de OSullivan entre la 24 y la 28 semanas de gestacin. 4. Repetir la glucemia basal entre la 24 y 28 semanas de gestacin. 5. Instaurar tratamiento diettico y controlar el perl glucmico.

378.

En un nio de 15 meses diagnosticado de enfermedad celaca, qu cereal, de los siguien-

-47-

EXAMEN ENARM 2/10

383.

Cul de las siguientes NO juega ningn papel en el screening y diagnstico de las malformaciones fetales: 1. Determinacin de beta-HCG en sangre materna. 2. Determinacin de alfafetoprotena en sangre fetal. 3. Cariotipo en sangre fetal. 4. Cariotipo del padre. 5. Ecografa.

387.

La causa ms frecuente de anomalas prenatales en el desarrollo humano es de origen: 1. 2. 3. 4. 5. Gentico. Ambiental. Infeccioso. Desconocido. Metablico.

388.

384.

Mujer de 32 aos, primigesta. En analtica del primer trimestre destaca en la serologa negatividad para Toxoplasma (IgM e IgG). Seale la respuesta INCORRECTA con respecto al manejo de esta paciente: 1. Est indicado realizar serologas de control a lo largo de toda la gestacin. 2. El riesgo de afectacin fetal est en relacin con el momento de infestacin materna. Si la madre padece la primoinfeccin en los ltimos meses de gestacin, el feto se afecta con menos frecuencia que al principio de sta. 3. La paciente debe seguir durante la gestacin una serie de recomendaciones higinicas (no consumir carne o derivados crudos o sin cocer, evitar el contacto con gatos, lavado de verduras...). 4. Si se produce afectacin fetal, uno de los signos sugerentes sera la presencia de calcicaciones intracraneales. 5. Como opciones teraputicas tenemos la espiramicina y la pirimetamina, sta ltima slo en el tercer trimestre.

Nia de 14 meses, que tras presentar un cuadro febril de vmitos y diarrea sanguinolenta hace 2 semanas, contina presentando diarrea lquida maloliente, ya sin sangre, con distensin abdominal, abundantes ruidos hidroareos e importante irritacin del rea perianal. Cul, entre las siguientes, sera la actitud ms adecuada? 1. Tratamiento antibitico oral. 2. Tranquilizar a los padres, pues se trata de una diarrea crnica inespecca que no afectar a su desarrollo. 3. Cambiar a frmula sin lactosa durante unas semanas. 4. Cambiar a frmula de hidrolizado de protenas. 5. Pautar rehidratacin con solucin de mantenimiento oral, sin variar la dieta.

389.

Qu actitud teraputica cree la ms indicada ante un neonato prematuro que tras una exanguinotransfusin presenta tetania con hipocalcemia que no responde a la administracin de calcio? 1. 2. 3. 4. 5. Administracin Administracin Administracin Administracin Administracin de de de de de fenobarbital. fsforo. magnesio. glucosa al 5%. vitamina D.

385.

Durante el primer trimestre de la gestacin pueden aparecer los siguientes sntomas EXCEPTO uno. Sealelo: 1. 2. 3. 4. 5. Reujo gastroesofgico. Estreimiento. Disuria. Aumento del apetito. Tensin mamaria.

390.

Un nio de 3 aos diagnosticado de Tetraloga de Fallot nos lo traen porque le encuentran ms irritable, ms ciantico y con polipnea. De todas las siguientes medidas teraputicas, cul NO estara indicada? 1. 2. 3. 4. 5. Decir al nio que se coloque de cuclillas. Administrar oxgeno. Inyectar morna subcutnea. Administrar bicarbonato. Iniciar digitalizacin.

386.

Acude una mujer de 28 aos a su consulta para recoger el resultado de una citologa, que resulta tener clulas compatibles con un SIL de bajo grado. Se realiza una colposcopia que demuestra un rea que no capta yodo en la tincin de Schiller. Tras realizar una biopsia de dicha rea se conrma el diagnstico. Cul sera la actitud teraputica ms adecuada? 1. Destruccin amplia de dicho rea por crioterapia o termocoagulacin. 2. Conizacin. 3. Histerectoma simple. 4. Vigilancia peridica. 5. Histerectoma radical.

391.

Seale la trada clnica ms sugestiva de toxoplasmosis congnita: 1. Hidrocefalia, calcicaciones intracerebrales y coriorretinitis. 2. Microcefalia, calcicaciones intracraneales y prpura trombocitopnica. 3. Cataratas, lesiones seas estriadas y sordera. 4. Lesiones mucocutneas, osteocondritis y periostitis. 5. Lesiones cutneas vesiculosas, queratoconjuntivitis y meningoencefalitis.

-48-

EXAMEN ENARM 2/10

392.

Cul de las siguientes vacunas NO es de virus atenuados? 1. 2. 3. 4. 5. Sarampin. Rubola. Parotiditis. Polio tipo Sabin. Polio tipo Salk.

4. Es incorrecta porque la GH no acta sobre el crecimiento en la etapa prenatal y neonatal. 5. Otra de las acciones de la GH es estimular la sntesis de protenas y lpidos. 398. Seale la armacin correcta sobre la enfermedad de injerto contra husped: 1. Es un efecto secundario de la terapia inmunosupresora. 2. Los linfocitos T del donante atacan a los tejidos del receptor. 3. Slo se produce en los trasplantes renales. 4. Los linfocitos B del injerto (donante) fabrican grandes cantidades de anticuerpos contra el HLA del receptor desarrollando una enfermedad autoinmune tpica. 5. El tratamiento consiste en retirar los inmunosupresores y realizar plasmafresis. 399. Cul es la fractura de estrs ms frecuente? 1. 2. 3. 4. 5. 400. Cuello de fmur. Tibia. Metatarsianos. Clavcula. Metacarpianos.

393.

Seale qu patologa NO asocia fontanela grande en el recin nacido: 1. 2. 3. 4. 5. Acondroplasia. Sndrome de Down. Hidrocefalia. Craneosinostosis. Hipotiroidismo.

394.

En la enfermedad sea de Paget se pueden observar todas las complicaciones que se citan EXCEPTO una: 1. 2. 3. 4. 5. Fracturas patolgicas. Policitemia. Impresin basilar. Compresin de pares craneales. Degeneracin maligna.

395.

A qu edad coge el lactante una bolita con ayuda del movimiento de pinza? 1. 2. 3. 4. 5. A A A A A los los los los los 10 meses. 6 meses. 8 meses. 5 meses. 15 meses.

El tumor cerebral ms frecuente en la infancia es: 1. 2. 3. 4. 5. Ependimoma. Meningioma. Astrocitoma. Glioblastoma. Papiloma del plexo coroideo.

396.

El dolor agudo escrotal en un nio de 12 aos requerir ciruga de urgencia en caso de: 1. 2. 3. 4. 5. Existir bacteriuria. Asociarse a ebre. Asociarse a exudacin uretral. Sospecharse torsin testicular. Asociarse a hematuria.

397.

La GH es una hormona sintetizada y liberada por la adenohipsis necesaria para el crecimiento lineal normal desde el momento del nacimiento, actuando indirectamente a travs de los IGF. Qu aadira a esta armacin? 1. Es errnea porque la GH acta directamente estimulando el crecimiento. 2. Adems de esto la GH posee varios efectos metablicos, entre ellos antagoniza la accin de la insulina actuando como hormona contrarreguladora fundamental, ejerciendo un efecto primario y a corto plazo en situaciones de hipoglucemia. 3. Los IGF son factores similares a la insulina sintetizados fundamentalmente en el cartlago de crecimiento y en el rin.

-49-

EXAMEN ENARM 2/10

IMAGEN 1

IMAGEN 2

-50-

EXAMEN ENARM 2/10

IMAGEN 3

IMAGEN 4

-51-

EXAMEN ENARM 2/10

IMAGEN 5

IMAGEN 6

-52-

EXAMEN ENARM 2/10

IMAGEN 7

IMAGEN 8

-53-

EXAMEN ENARM 2/10

IMAGEN 9

IMAGEN 10

-54-

EXAMEN ENARM 2/10

IMAGEN 11

IMAGEN 12

-55-

EXAMEN ENARM 2/10

IMAGEN 13

IMAGEN 14

-56-

EXAMEN ENARM 2/10

IMAGEN 15

IMAGEN 16

-57-

EXAMEN ENARM 2/10

IMAGEN 17a

IMAGEN 17b

IMAGEN 18

-58-

EXAMEN ENARM 2/10

IMAGEN 19

IMAGEN 20

-59-

EXAMEN ENARM 2/10

IMAGEN 21

IMAGEN 22

-60-

EXAMEN ENARM 2/10

IMAGEN 23

IMAGEN 24

-61-

Das könnte Ihnen auch gefallen